Jara Power Notes

December 30, 2017 | Author: Jacob Andrews | Category: Parol Evidence Rule, Evidence (Law), Evidence, Pleading, Judicial Notice
Share Embed Donate


Short Description

Download Jara Power Notes...

Description

20112012 Layno, Ian Rabajante, Diory Morana, Hansel Reyes, Harly Suyat, Kat Baraoidan, Kimberly

[JARA POWER NOTES ON EVIDENCE] This is merely a compilation of questions asked by Dean Jara for Evidence in his 2011-2012 class. The writers guarantee the integrity of all the questions and the materials used in the making of this recitation reviewer. However, there is no guarantee that these will be the same set of questions he will ask on the subsequent semesters/years. Read this together with Riano, Regalado, Moran, Memaid, and Jara Notes.

[JARA POWER NOTES ON EVIDENCE]

2011-2012

JARA RECITATION NOTES FOR EVIDENCE PRELIMINARY MATTERS

It refers to a denial of the material facts or facts alleged in the pleading of the claimant essential to his cause of action.

What is Evidence? Evidence is the means, sanctioned by these rules, of ascertaining in a judicial proceeding the truth respecting a matter of fact. Can Evidence be used to answer questions of fact? How about questions of law? As provided for by the definition, Evidence can be used to answer ONLY questions of fact, and not questions of law. Why cannot we use Evidence to answer disputes of law? Under our legal system, it is conclusively presumed that everybody knows the law. Ignoratia legis non-excusat. All parties to the dispute must know the law applicable in the case. There could be o dispute between the parties and even the court itself concerning the truth about a matter of law. Distinguish Questions Questions of Law

of

Fact

Define a specific denial.

from

Question of fact (also known as a point of fact) is a question which must be answered by reference to facts and evidence, and inferences arising from those facts. On the other hand, a question of law is a question which must be answered by applying relevant legal principles, by an interpretation of the law.

A defendant must specify each material allegation of fact the truth of which he does not admit and, whenever practicable, shall set forth the substance of the matters upon which he relies to support his denial. Where a defendant desires to deny only a part of an averment, he shall specify so much of it as is true and material and shall deny only the remainder. Where a defendant is without knowledge or information sufficient to form a belief as to the truth of a material averment made to the complaint, he shall so state, and this shall have the effect of a denial. (Sec.10, Rule 8) Can there be a question of fact when a party raises an affirmative defense? NO. When a party raises an affirmative defense, he in effect, hypothetically admits the material allegations of his opponent in his pleadings. In a case where a party defaults, is there still a need to present evidence? YES. The court should still require the nondefaulting party to present his evidence. There is still a need to prove the allegations made in the complaint. In every case that requires adjudication, is there a need to present evidence at all times?

Is the issue on jurisdiction a question of fact or law?

No. There are instances when Evidence is not required to be presented such as the following:

It is a question of law. Jurisdiction pertains to the power of the Courts to decide cases. It is vested by substantive law. Any issue relating to it is a legal question.

1. When no factual issue exists in a case. 2. Where the case presents only a question of law. 3. When the pleadings in a civil case do not tender an issue of fact. 4. Evidence may also be dispensed with by agreement of the parties. 5. Matters of judicial notice (See Rule 129). 6. Matters judicially admitted (See Rule 129).

When does a question of fact arise? When the issues are already joined. When does that joinder happen? Upon the filing of an answer which contains a specific denial as to the existence of a certain fact.

Layno. Rabajante. Morana. Reyes. Suyat. Baraoidan|

2

2011-2012

[JARA POWER NOTES ON EVIDENCE]

Are evidentiary rules also found outside Rules 128-133?

True or False. Probandum is always determined by the allegations in the pleadings.

YES. FALSE. Probandum may also be determined by the pre-trial order and it may change during the course of the trial (see Sec. 5, Rule 10).

Illustration? Note: Example ni Jara ito. Supposing in a complaint for sum of money, the plaintiff alleges that the defendant is indebted to him for P10 million pesos. Upon the presentation of evidence, the plaintiff presents to the court a promissory note evidencing the indebtedness of the defendant. The note however indicates that such indebtedness amounts to P30 million pesos. May the court admit aforementioned evidence even if the original complaint only alleged P10 million? YES. Under Sec. 5, Rule 10 (Amendment to conform or to authorize presentation of evidence). You see, this is an evidentiary rule outside Rules 128-133 (This was a total mind-fuck. Please take note of this example of his). What is Factum Probandum and Factum Probans? Factum probandum is the ultimate fact or the fact sought to be established. Factum Probans pertains to the evidentiary fact or the facts by which the Probandum is established. The former refers to the proposition, while the latter refers to the materials which establish that proposition. Does probandum exist in a case where a party defaults? YES. Party still needs to present evidence exparte. A prosecutor files in the court an information for homicide. Is there probandum already? NO. Because the accused has yet to be arraigned. In criminal cases, the arraignment of the accused determines whether or not there exists a probandum. If the accused pleads guilty to a crime, does that negate the existence of probandum? Not in all instances. Probandum is still necessary when the case involves a heinous crime.

Can probandum change easily in a criminal case? No. probandum cannot be changed the same way it can be changed in a criminal case. There is a necessity to amend the pleadings according to the Rules on Criminal Procedure. When do we apply the Rules on Evidence? The rules of evidence, being part of the Rules of Court, apply only to judicial proceedings, subject to inapplicable cases in Sec. 4, Rule 128. Can we apply Proceedings?

evidence

in

Summary

Yes. Summary proceedings partake the nature of judicial proceedings. Being the case, we can definitely apply the Rules on Evidence in such. Do we present Evidence Proceedings? If so, what evidence?

in Summary then is our

YES. Evidence is still presented by virtue of the pleadings and affidavits submitted before the court. They partake the nature of documentary evidence and they serve to prove an issue of fact provided that they are admissible pursuant to law. How about in Quasi-Judicial Proceedings? Yes. The same apply by analogy whenever practicable and convenient except where the governing law on that particular proceeding specifically adopts the rules of evidence in the Rules of Court. Distinguish Proof from Evidence. Proof is not the evidence itself. There is proof only because of evidence. It is merely the probative effect of evidence and is the conviction or persuasion of the mind resulting from a consideration of evidence. On the other hand,

Layno. Rabajante. Morana. Reyes. Suyat. Baraoidan|

3

2011-2012

[JARA POWER NOTES ON EVIDENCE]

Evidence is the medium by which a fact is proved or disproved. What are the different classifications of evidence? According to Form: 1. Object: evidence that is directly addressed to the senses of the court and consists of tangible things exhibited or demonstrated in open court, in an ocular inspection, or in a designated place. 2. Documentary: evidence supplied by written instruments or derived from conventional symbols, such as letters, by which ideas are represented on material substances. 3. Testimonial: submitted to the court through the testimony or the depositon of a witness. As to ability to establish a fact in dispute: 1. Direct: proves a fact in dispute without any aid of any inference or presumption. 2. Circumstantial: proof of fact or facts which taken singly or collectively, the existence of a fact in dispute may be inferred as a necessary or a probable consequence. As to Probative Value: 1. Prima Facie: that which standing alone unexplained, is sufficient to establish the proposition affirmed. 2. Conclusive: Evidence which is incontrovertible. 3. Corroborative: additional evidence of a different kind and character from that already given tending to prove the same point. 4. Cumulative: additional evidence of the same kind and character proving the same fact. As to weight and acceptability: 1. Primary: evidence which affords the greatest certainty of the fact in question. 2. Secondary: evidence which is inferior to primary evidence.

As to Quality: 1. 2. 3. 4. 5.

Relevant Material Admissible Credible Competent

What are the Axioms of Admissibility? Axiom of Relevance: None but the facts having a rational probative value are admissible. Axiom of Competence: Facts having a rational probative value are admissible unless some specific rule prohibits their admission. What are the different classes of admissibility according to jurisprudence? 1. Multiple Admissibility of Evidence – where the evidence is relevant and competent for two or more purposes, such evidence may be admitted for any or all purposes which it is offered provided it satisfies all the requirements of law for the admissibility therefor. 2. Conditional Admissibility of Evidence – where evidence initially offered appears to be immaterial or irrelevant unless it is connected with facts to be subsequently proved, such evidence may be admitted under the condition that the subsequent facts shall be proven; otherwise, they shall be stricken out from the record. 3. Curative Admissibility of Evidence – Where improper evidence was admitted against the opposition of another party, he may be allowed to contradict it with similar improper evidence. WHAT NEED NOT BE PROVEN What are some matters that do not require any evidence? 1. Where no factual issue exists in a case. 2. Where the case presents only a question of law. 3. When the pleadings in a civil case do not tender an issue of fact. 4. When there is an agreement made by the parties (usually done in the pre-trial). 5. When courts have taken judicial notice of such matters.

Layno. Rabajante. Morana. Reyes. Suyat. Baraoidan|

4

2011-2012

[JARA POWER NOTES ON EVIDENCE]

What do you mean by Judicial Notice?

ambassador, or a foreign public official, and para sigurado, idaan mo na din sa DFA).

It is the cognizance of certain facts which judges may properly take and act upon without proof. Why? I thought facts must be proven in court? Yes. That is the general rule. However, there are some matters, on the basis of expediency and convenience, which do not require proof. The need for evidence is set aside by the rules. What are matters subject of Judicial Notice? 1. Existence and territorial extent of states; 2. Political history, forms of government, and symbols of nationality; 3. Law of nations 4. Admiralty and maritime courts of the world and their seals; 5. Political constitution and history of the Philippines; 6. Official acts of the legislative, executive, and judicial departments of the Philippines; 7. Laws of nature; 8. Measure of time; and 9. Geographical divisions. What is the Presumption?

doctrine

of

Processual

The foreign law, whenever applicable, should be proved by the proponent thereof, otherwise, such law shall be presumed to be exactly the same as the law of the forum. Is it not the duty of the Courts to apply the Local Law? It is. However, there are certain transactions and legal acts that are bound by laws of foreign jurisdictions. It is important that courts respect the governing laws over such acts. However, the existence of such laws must be proven as a matter of fact. How do you then prove foreign law? As for foreign statutes and issuances, you prove the foreign law through acquiring a copy of the said document which is certified by the agent of the said country through the DFA (As per Jara‘s lecture, get a copy of the law abroad or in the embassy, have it certified through a consul,

Do we still need certification that the said foreign law exists even though the existence of such has been stipulated by both parties? The general rule is that the foreign law must still be proven as a matter of fact. However, in the case of PCIB vs. Escolin, the Supreme Court held that where the foreign law is within the actual knowledge of the court such as when the law is generally well known, had been ruled upon in previous cases before it and none of the parties claim otherwise, the court may take judicial notice of the foreign law. In essence, the Escolin case gives the requisites for the exception: 1. Actual knowledge by the Court; 2. Foreign law is generally well known; 3. Stipulation by the parties. What if the foreign law is Common Law? How do you prove such creature? Common law (also known as case law or precedent) is law developed by judges through decisions of courts and similar tribunals (as opposed to statutes adopted through the legislative process or regulations issued by the executive branch). You prove such by getting a copy of the foreign court‘s decision and having the same certified in the like manner as you certify foreign statutes. Is Judicial Notice absolute according to the Rules? No. There are some matters that require hearing before courts can take judicial notice. When does it require hearing? 1. During trial, the court, on its own initiative, or on request of a party, may announce its intention to take judicial notice of any matter and allow the parties to be heard thereon. 2. After the trial, and before judgment or on appeal, the court may take judicial notice of any matter and allow the parties to be heard thereon if such matter is decisive of a material issue in the case.

Layno. Rabajante. Morana. Reyes. Suyat. Baraoidan|

5

2011-2012

[JARA POWER NOTES ON EVIDENCE]

What are Judicial Admissions? It is an admission, verbal or written, made by a party in the course of the proceedings. Note: for an extensive discussion admissions, please see the memory aid.

Usually in cases where there is an amendment of pleadings, a judicial admission is transformed into an extrajudicial admission. Will the same hold true in the problem above?

on Yes. Party must then raise such admissions as evidence.

Lather. Rinse. Repeat. May a complaint admission?

contain

a

judicial

Yes, if there is an allegation that is contrary to the interests of the complainant. In evidence, such admission must be DISADVANTAGEOUS to the one who made the complaint (Jara statement). Are admissions made by a counsel binding upon his client? Generally, Yes. Unless of course the lawyer was patently stupid. Note: The function of the rule that negligence or mistake of counsel in procedure is imputed to and binding upon the client, as any other procedural rule, is to serve as an instrument to advance the ends of justice. When in the circumstances of each case the rule desert[s] its proper office as an aid to justice and becomes its great hindrance and chief enemy, its rigors must be relaxed to admit exceptions thereto and to prevent a manifest miscarriage of justice. The court has the power to except a particular case from the operation of the rule whenever the purposes of justice require it [Aguilar v. CA, 320 Phil. 456 (1995)]. What if because of the stupidity of the lawyer, he made a general denial of all the allegations? Normally, the case would be ripe for judgment on the merits. However, the client may still amend his pleading as a matter of right especially if there is no reply yet.

What happens to the judicial admissions contained in the pleadings which are subsequently amended? An amended pleading supersedes the pleading that it amends. However, admissions in superseded pleadings may be received in evidence against the pleader; and claims or defenses alleged therein not incorporated in the amended pleading shall be deemed waived (Sec.8, Rule 10). Such admissions, however, are considered extra-judicial admissions. Having been amended, the original complaint lost its character as a judicial admission, which would have required no proof, and became merely an extrajudicial admission, the admissibility of which, as evidence, required its formal offer (Torres vs. CA, L-37420-21, July 31, 1984). In appealed cases, may there be a verbal judicial admission? Yes. In cases for a petition for new trial. What the doctrine of adoptive admission? An adoptive admission is a party‗s reaction to a statement or action by another person when it is reasonable to treat the party‗s reaction as an admission of something stated or implied by the other person (Estrada v. Desierto, G.R. Nos. 146710-15). In this case, Estrada‗s admission of his resignation as President was based on the diary of Angara. Supposing evidence was presented tending to prove a substantially different (usually greater) claim which was not subject of a previous admission by the adverse party during the pre-trial, may the court admit such evidence? YES. Rule 10, Sec. 5.

Layno. Rabajante. Morana. Reyes. Suyat. Baraoidan|

6

2011-2012

[JARA POWER NOTES ON EVIDENCE]

Does information contain admissions in so far as People of the Philippines are concerned?

since we find that far too much of the space in the transcript is taken up with the record of petty skirmishes in court resulting from objections over the admission of evidence.

No. Issue: WON the technical rules of evidence may be relaxed in admitting evidence.

Is there a need to wait for a response? No. Is there a need for a judicial admission to be prejudicial to the admitter? No. The self-serving rule which prohibits the admission or declaration of a witness in his favor applies only to extra-judicial admissions. Self-serving statements, even if made in the complaint, are admissible because the claimant or the witness making it may be cross-examined on such statements. However, whether it will be credible or not, is a matter of appreciation on the part of the court. What was the case of Prats vs. Phoenix? Facts: This action was instituted in the Court of First Instance of the City of Manila by Prats & Co., a mercantile partnership, for the purpose of recovering from the Phoenix Insurance Co., of Hartford, Connecticut, the sum of P117,800.60, with interest, by reason of a loss alleged to have been sustained by the plaintiff, on August 21, 1924, from a fire, it being alleged that said loss was covered by policy of insurance No. 600217, for the sum of P200,000, issued by the defendant company to the plaintiff. For answer, the defendant, Pheonix Insurance Co., admitted the insurance of the policy of insurance but, by way of special defense, alleged, among other things, that the fire in question had been set by the plaintiff, or with its connivance, and that the plaintiff had submitted under oath to the defendant a fraudulent claim of loss, in contravention of the express terms of the policy. The trial of the case covered a period of almost two years, in which fifty separate sessions were held, without counting the numeruos hearings upon the taking of the deposition of Francisco Prats, a partner in the plaintiff firm, whose testimony was taken at the instance of the defendant. Taken all together, the time thus consumed was out of all proportion to the difficulties of the case. An examination of the voluminous transcript reveals at least part of the reason for this inordinate consumption of time;

Held: Yes. In the course of long experience we have observed that justice is most effectivly and expenditiously administered in the courts where trivial objections to the admission of proof are received with least favor. The practice of excluding evidence on doubtful objection to its materiality or technical objection to the form of the questions should be avoided. In a case of any intricacy it is impossible for a judge of first instance, in the early stages of the development of the proof, to know with any certainty whether testimony is relevant or not; and where there is no indication of bad faith on the part of the attorney offering the evidence, the court may as a rule safely accept the testimony upon the statement of the attorney that the proof offered will be connected later. Moreover, it must be remembered that in the heat of the battle over which he presides a judge of first instance may possibly fall into error in judging of the relevancy of proof where a fair and logical connection is in fact shown. When such a mistake is made and the proof is erroneously ruled out, the Supreme Court, upon appeal, often finds itself embarrassed and possibly unable to correct the effects of error without returning the case for a new trial, -- a step which this court is always very loath to take. On the other hand, the admission of proof in a court of first instance, even if the question as to its form, materiality, or relevancy is doubtful, can never result in much harm to either litigant, because the trial judge is supposed to know the law; and it is its duty, upon final consideration of the case, to distinguish the relevant and material from the irrelevant and immaterial. If this course is followed and the cause is prosecuted to the Supreme Court upon appeal, this court then has all the material before it necessary to make a correct judgment. In this connection it should be remembered that many of the technical rules of evidence which are often invoked in our courts were originally worked out in England and the United States, where the jury system prevails. These rules were adopted for the purpose of keeping matter from juries which - it was supposed - might

Layno. Rabajante. Morana. Reyes. Suyat. Baraoidan|

7

2011-2012

[JARA POWER NOTES ON EVIDENCE]

unduly influence them in deciding on the facts. They have little pertinence to a system of procedure, like ours, in which the court is judge both of law and facts, and in which accordingly it is necessary for the court to know what the proof is before it rules upon the propriety of receiving it. Apart from these considerations is the circumstance mentioned above that the time consumed in the trial on such collateral points is generally many times greater than would be consumed if the questionable testimony should be admitted for what it is worth. What has been said above finds special relevancy in this case in view of the action of the trial court in refusing to consider the proof referred to in the opinion showing that the plaintiff, while engaged in assembling its stock, procured maritime insurance upon a fictitious importation of silk. We earnestly commend the maintenance of liberal practice in the admission of proof. Our examination of the case leads to the conclusion that the result reached by the trial court was correct. What happened in the case of Maceda? Facts: This is a criminal action for slight slander instituted in the justice of the peace court of Pasig, Rizal, against defendants Generoso Maceda and Corazon Maceda and which was dismissed on the ground that the offense had already prescribed. The offense was allegedly committed on July 21, 1940, and the action was filed on October 22, 1940, that is three months and one day after the supposed commission thereof. The private attorney for the offended party made an admission to the effect that no damages had been sustained by the latter. Issue: WON the offended party may be bound by the admission of his attorney.

Are there other terms for object evidence? 1. 2. 3. 4.

Real Physical Demonstrative Autoptic preference.

Are there rules on exclusion for object evidence? Section 1 of Rule 130 provides that when an object is relevant to the fact in issue, it may be exhibited to, examined or viewed by the court. Aside from being Relevant? Generally, there could be no rules for the exclusion of object evidence. Are there exceptions? 1. The exhibition of such object is contrary to public policy, morals or decency; 2. Observation of such evidence in an ocular inspection would result in delays, inconvenience, unnecessary expenses out of proportion to the evidentiary value of such object; 3. Such object evidence would be misleading or confusing; 4. The testimonial or documentary evidence already presented in curt already portrays the object in question as to render a view thereof unnecessary. 5. Those evidence acquired through illegal searches and seizures. What do you call to those evidence acquired through unlawful searches and seizures? Derivative Evidence.

Held: In the instant case, the supposed admission is denied. Besides, no attorney can waive his client's cause of action unless with the consent of the client, and, in the instance case, the admission attributed to the private prosecuting attorney is not alleged to have been made with the offended party's consent. OBJECT EVIDENCE

What are the categories of Object Evidence? 1. Unique – objects that have readily identifiable marks. 2. Objects made unique – objects that are readily made identifiable. 3. Non-unique – objects with no identifying marks and cannot be marked.

What is Object Evidence? Objects as evidence are those addressed to the senses of the court.

Layno. Rabajante. Morana. Reyes. Suyat. Baraoidan|

8

2011-2012

[JARA POWER NOTES ON EVIDENCE]

How does the court conduct an ocular inspection? The court goes to the place where the object evidence is located, whenever the said object cannot be brought to the chambers of the court because it is an immovable or moving such object would cause undue inconvenience. Is ocular inspection part of the judicial proceedings? Yes. It is part of the trial. Inasmuch as evidence is thereby being received, such inspection should be made in the presence of the parties or at least with previous notice to them of the time and place set for the view. Is the judge always needed to be present in the ocular inspection? No. There are certain evidence-gathering procedures that do not require the presence of the judge such as those proceedings conducted by commissioners.

there was present as in interested, spectator, another witnesses, who, without being sworn, could tell the truth and nothing but the truth. This was the colt. The colt was separated from the mare of the complaining witness and turned loose; it at once went back to this mare. The colt was then taken to the mare of the accused; but showed its dislike for the mare and tried to find the mare of the complaining witness. Another colt was placed near the mare of the complaining witness; thereupon the mare and that colt both resisted. This was a practical demonstration worthy of a Solomon by which the colt was able to testify by manifesting all the signs of the young, whether human or not, on finding a long lost mother. In the case of Tegrado, why did the court gave reference to King Solomon? A Solomonic approach was done in the case of Tegrado because of the way the court arrived with a proof – through the use of demonstrative evidence, in observing the demeanor of the horse in order to determine its true owner.

Can there be a practical mechanism to allow the inspection while dispensing the need for the presence of the judge?

What was the object evidence in Tegrado?

I just answered your question bitch.

What happened in the case of People vs. Gutierrez?

The court observed the horse and its demeanor.

What is the case of US vs. Tegrado? A colt valued at P34 was stolen from Valeriano Blanca. It was subsequently found in the possession of Agapito Partolan. The latter testified that he bought the animal from Zacarias Tegrado, the accused. The accused, however, claimed that the colt was raised from a mare belonging to him and then sold to Partolan. Identification of the colt to determine if its mother was a mare belonging to the complainant Valeriano Blana or if its mother was a mare belonging to the accused Zacarias Tegrado is, therefore, the determining factor. The colt was identified by a number of witnesses as the property of Blanca. Other witnesses testified to having seen the colt following a mare belonging to the accused. Whom shall we believe? We could, of course, rest our conclusion on the findings of the trial court. We could, in addition, point out grave discripancies in the testimony of the witnesses for the defense, which argues against its reliability. But

At around 4:45 p.m. on September 12, 2002, the police station of Ramos, Tarlac acting on a tip regarding a shabu transaction (drug-pushing) taking place somewhere in Purok Jasmin, Poblacion Norte, dispatched a three-man team composed of PO3 Romeo Credo, P/Insp. Napoleon Dumlao, and SPO1 Restituto Fernandez to the place mentioned. Arriving at the target area, the three noticed Fernando and one Dennis Cortez under a santol tree handing plastic sachets containing white crystalline substance to certain individuals. At the sight of the police officers, Fernando and the others scampered in different directions. After a brief chase, however, one of the three police operatives caught up with and apprehended Fernando, then carrying a bag. Without a trace of equivocation, the RTC and later the CA held that the prosecution had discharged the burden of proving all the elements of the crime charged. Since Fernando was caught carrying the incriminating bag after

Layno. Rabajante. Morana. Reyes. Suyat. Baraoidan|

9

2011-2012

[JARA POWER NOTES ON EVIDENCE]

the police had been tipped off of drug pushing in the target area, any suggestion that he was not in actual possession or control of the prohibited drug hidden in the area would be puny. Thus, ownership of the bag is truly inconsequential. We emphasize at this juncture that in no instance did Fernando intimate to the trial court that there were lapses in the safekeeping of the seized items that affected their integrity and evidentiary value. He, thus, veritably admits that the crystalline substance in the sachets found in his bag was the same substance sent for laboratory examination and there positively determined to be shabu and eventually presented in evidence in court as part of the corpus delicti. In other words, Fernando, before the RTC and the CA, opted not to make an issue of whether the chain of custody of the drugs subject of this case has been broken. This disposition on the part of Fernando is deducible from the August 18, 2005 Order of the trial court, pertinently saying, ―[The] Acting Provincial Prosecutor x x x and Atty. Emmanuel Abellera, counsel de officio of the accused manifested that the chain of custody of the searched illegal drug or shabu is admitted.‖ As a mode of authenticating evidence, the chain of custody rule requires that the presentation of the seized prohibited drugs as an exhibit be preceded by evidence sufficient to support a finding that the matter in question is what the proponent claims it to be. This would ideally cover the testimony about every link in the chain, from seizure of the prohibited drug up to the time it is offered in evidence, in such a way that everyone who touched the exhibit would describe how and from whom it was received, to include, as much as possible, a description of the condition in which it was delivered to the next link in the chain. What was the object evidence in the case of Gutierrez? The seized drugs. What is the Doctrine of the Chain of Custody? Chain of Custody means the duly recorded authorized movements and custody of seized drugs or controlled chemicals or plant sources of dangerous drugs or laboratory equipment of each stage, from the time of seizure/confiscation

to receipt in the forensic laboratory to safekeeping to presentation in court for destruction. Such record of movements and custody of seized item shall include the identity and signature of the person who held temporary custody of the seized item, the date and time when such transfer of custody were made in the course of safekeeping and used in court as evidence, and the final disposition (Section 1, DDB Regulation No. 1, Series of 2002). How is the process conducted? Under Sec. 21 of the Comprehensive Dangerous Drugs Acts of 2002: 1. The apprehending team having initial custody and control of the drugs shall, immediately after seizure and confiscation, physically inventory and photograph the same in the presence of the accused or the person/s from whom such items were confiscated and/or seized, or his/her representative or counsel, a representative from the media and the DOJ, and any elected public official who shall be required to sign the copies of the inventory and be given a copy thereof; 2. Within 24 hours upon confiscation/seizure of dangerous drugs, plant sources of dangerous drugs, controlled precursors and essential chemicals, as well as instruments/paraphernalia and/or laboratory equipment, the same shall be submitted to the PDEA Forensic Laboratory for a qualitative and quantitative examination. 3. A certification of the forensic laboratory examination results, which shall be done under oath by the forensic laboratory examiner, shall be issued within 24 hours after the receipt of the subject item/s: Provided, that when the volume of the dangerous drugs, plant sources of dangerous drugs, and controlled precursors and essential chemicals does not allow the completion of testing within the time frame, a partial laboratory examination report shall be provisionally issued stating therein the quantities of dangerous drugs still to be examined by the forensic laboratory: Provided, however, that a final certification shall be issued on the

Layno. Rabajante. Morana. Reyes. Suyat. Baraoidan|

10

2011-2012

[JARA POWER NOTES ON EVIDENCE]

completed forensic laboratory examination on the same within the next 24 hours; 4. After the filing of the criminal case, the Court shall, within 72 hours, conduct an ocular inspection of the confiscated, seized and/or surrendered dangerous drugs, plant sources of dangerous drugs, and controlled precursors and essential chemicals, including the instruments/ paraphernalia and/or laboratory equipment, and through the PDEA shall within 24 hours thereafter proceed with the destruction or burning of the same, in the presence of the accused or the person/s from whom such items were confiscated and/or seized, or his/her representative or counsel, a representative from the media and the DOJ, civil society groups and any elected public official. The Board shall draw up the guidelines on the manner of proper disposition and destruction of such item/s which shall be borne by the offender: Provided, that those item/s of unlawful commerce, as determined by the Board, shall be donated, used or recycled for legitimate purposes: Provided further, that a representative sample, duly weighed and recorded is retained; 5. The Board shall then issue a sworn certification as to the fact of destruction or burning of the subject item/s which, together with the representative sample/s in the custody of the PDEA, shall be submitted to the court having jurisdiction over the case. In all instances, the representative sample/s shall be kept to a minimum quantity as determined by the Board; and 6. The alleged offender or his/her representative or counsel shall be allowed to personally observe all of the above proceedings and his/her presence shall not constitute an admission of guilt. In case the said offender or accused refuses or fails to appoint a representative after due notice in writing to the accused or his/her counsel within 71 hours before the actual burning or destruction or the evidence in question, the SOJ shall appoint a member of the PAO to represent the former;

7. After the promulgation and judgment in the criminal case wherein the representative sample/s was presented as evidence in court, the trial prosecutor shall inform the Board of the final termination of the case and, in turn, shall request the court for leave to turn over the said representative sample/s to the PDEA for proper disposition and destruction within 24 hours from receipt of the same. Is it necessary that the whole amount of drugs must be presented in evidence? No. Only a sample amount as determined by the DDB may be presented to the court. How do you then prove the existence of such amount? Through an initial certification and a subsequent ocular inspection done by the court hearing the case. Is that not substitution? No. Since a sample was taken from the very objects that were seized by the authorities themselves. Besides, to require the presentation of all the amount/volume of evidence before the court would only cause great inconvenience. You do not want to destroy Judge Wagan‘s mood by wasting her time looking at the stacks of marijuana and accounting for their volume. Judge Wagan can be very angry. And you would not like it when she‘s angry. Are pictures in drug cases sufficient? No, because the narcotic substance is the very corpus delicti of the crime. Is failure to comply strictly with the mandate of Sec. 21 of R.A. No. 9165 (pertaining to the custody and disposition of confiscated, seized, and surrendered dangerous drugs, etc.) fatal to the prosecution? No. Non-compliance by the apprehending/buybust team with Section 21 is not fatal as long as there is justifiable ground, and as long as the integrity and the evidentiary value of the confiscated/seized items are properly preserved by the apprehending officer/team. Its noncompliance will not render an accused‘s arrest

Layno. Rabajante. Morana. Reyes. Suyat. Baraoidan|

11

2011-2012

[JARA POWER NOTES ON EVIDENCE]

illegal or the items seized/confiscated from him inadmissible. What is of utmost importance is the preservation of the integrity and the evidentiary value of the seized items, as the same would be utilized in the determination of the guilt or innocence of the accused. Crucial in proving the chain of custody is the marking of the seized driugs or other related items immediately after thet are seized from the accused. Marking after seizure is the starting point in the custodial link, thus, it is vital that the seized contraband are immediately marked because succeeding handlers of the specimens will use the markings as reference. The marking of the evidence serves to separate the marked evidence from the corpus of all other similar or related evidence from the time they are seized from the accused until they are disposed of at the end of criminal proceedings, obviating switching, ―planting,‖ or contamination of evidence (People of the Philippines v. Allen Udtojan Mantalaba, G.R. No. 186227).

Is it not the right of the plaintiff to choose the type of object evidence he may desire for as long as it is relevant? Yes. That is the GENERAL RULE. However, it must still be subject to the common norms of public policy, decency and morality. What is your legal basis? Jara: court decisions have established certain grounds to refuse to admit such evidence. (He did not cite any case. Up to you to look for them). Let us take for example a special proceeding. A person wants to change the sex indicate in the entry in his birth certificate from male to female. Mali daw ang nalagay. Dapat daw babae siya. Can the plaintiff undress before the court? No. Again, it will offend the norms of decency. It would be better if were done in closed chambers.

Is the class card an object evidence? It depends. If we pertain solely to the paper itself, the class card itself without having to look into the contents of the same, it can be said that the class cards partake the nature of object evidence. On the other hand, if there is a need to view the contents written on the class cards, they partake the nature of documentary evidence. Supposing there was an action for damages arising from a quasi-delict on account of a vehicular accident. In the complaint, the plaintiff alleged that he figured in the collision and in the process, his arm got amputated. Can the plaintiff bring to the court the amputated arm? Nope. for the following reasons (According to Morana‘s recollection of Jara‘s explanation):

What is Demonstrative Evidence? How does one test its admissibility? Demonstrative evidence is not the actual thing but it is referred to as ―demonstrative‖ because it represents or demonstrates the real thing. It is not strictly ―real‖ evidence because it is not the very thing involved in the case. A map, a diagram, a photograph and a model, fall under this category. The admissibility of this type of evidence largely depends on laying the proper foundation for the evidence. The rule boils down to one basic question: Does the evidence sufficiently and accurately represent the object it seeks to demonstrate or represent? If it does, the evidence would be admissible DOCUMENTARY EVIDENCE

1. Kadiri. It is offensive to the senses and thus, will violate the norms of decency in court proceedings. 2. Madaya. Can you imagine if you are the judge and while you are writing your decision, the amputated arm is right before your eyes? It will be prejudicial to the defendant in that case.

BEST EVIDENCE RULE What is Documentary Evidence? Documentary Evidence consists of writings or any material containing letters, words, numbers, figure, symbols or other modes of written expressions offered as proof of their contents.

Layno. Rabajante. Morana. Reyes. Suyat. Baraoidan|

12

2011-2012

[JARA POWER NOTES ON EVIDENCE]

For what purpose do we offer documentary evidence?

So if I present a photocopy of a certain document, is it automatically excluded because of the Best evidence Rule?

In order to prove the contents of the writing. What is the Best Evidence Rule? The term ―best has nothing to do with the degree of its probative value in relation to other types of evidentiary rules. It is not intended to mean the ―most superior evidence. More accurately, it is the ―original document rule‖ or the ―primary evidence rule. This rule applies to documentary evidence only, or a document presented as proof of its contents. It does not apply where there is no bona fide dispute on the contents of documents and no useful purpose would be served by its production. Simply stated, when the subject of inquiry is the contents of a document, no evidence shall be admissible other than the original document itself. Is the Rule Absolute? No. The Best Evidence Rule admits several exceptions: 1. When the original has been LOST or DESTROYED, or cannot be PRODUCED in court, without bad faith on the part of the offeror. 2. When the original is in the CUSTODY or under the CONTROL of the party against whom the evidence is offered, and the latter fails to produce it after reasonable notice. 3. When the original consists of NUMEROUS ACCOUNTS or other documents which cannot be examined in court without great loss of time and the fact sought to be established from them is only the general result of the whole. 4. When the original is a PUBLIC RECORD in the custody of a public officer or is recorded in a public office.

NO. If the presentation of the photocopied document is only for the purpose of proving the existence, execution, or the delivery of the said photocopied document without any reference as to its terms, such evidence may still be admissible. Under the best evidence rule, the original document must be produced whenever its contents are the subject of inquiry. A photocopy, being mere secondary evidence, is not admissible unless it is shown that the original is unavailable. Before a party is allowed to adduce secondary evidence to prove the contents of the original, the offeror must prove the following: 1. The existence or due execution of the original; 2. The loss and destruction of the original or the reason for its non-production in court; and 3. On the part of the offeror, the absence of bad faith to which the unavailability of the original can be attributed. The correct order of proof is as follows: existence, execution, loss, and contents Does the “Best Evidence Rule” apply absolutely in all cases a document is presented as an evidence in court? NO. With respect to documentary evidence, the best evidence rule applies only when the contents of such document is the subject of inquiry. Where the issue is only as to whether such a document was actually executed, or exists, or on the circumstances relevant to or surrounding its execution, the best evidence rule does not apply and testimonial evidence is admissible OK. I present the original of a document. Is it automatically admitted?

Why is there such a rule? 1. To prevent fraud; 2. To exclude uncertainties in the contents of a document

No. It has to undergo the process of authentication, except if the document is a public document.

Layno. Rabajante. Morana. Reyes. Suyat. Baraoidan|

13

2011-2012

[JARA POWER NOTES ON EVIDENCE]

PAROL EVIDENCE RULE

How do you determine whether or not the subject of an oral agreement is separate and distinct from the subject of the writing?

What is the Parol Evidence Rule? GENERAL RULE: When the terms of an agreement have been reduced to writing, it is considered as containing all the terms agreed upon and there can be, between the parties and successors in interest, no evidence of such terms other the contents of the written agreement. EXCEPTION: a party may present evidence to modify, explain or add to the terms of the written agreement if he puts in issue in his pleading: 1. The failure of the written agreement to express the true intent of the parties thereto; 2. The intrinsic ambiguity, mistake or imperfection in the written agreement; 3. The validity of the written agreement; 4. The existence of other terms agreed to by the parties or their successors in interest after the execution of the written agreement. The term ―agreement‖ includes wills. What is Parol Evidence?

STEPS: 1. Determine first what is the whole subject intended by the parties to be covered by such writing. This question may be determined from the contract itself, in light of the subject-matter with which it deals and of the circumstances attending its execution. 2. Ascertain the subject of the oral agreement offered to be proved. To this effect, the parol evidence may be admitted provisionally. 3. A comparison should be made between the writing and the oral negotiation and from that comparison it may be seen whether or not the subject of the writing is separate and distinct from that of the oral negotiation. 4. If the subject of the oral negotiation is not so closely connected with the subject of the writing as to form part and parcel of it, then parol evidence is admissible. Distinguish Parol Evidence rule from the Best Evidence Rule.

Parol evidence is any evidence aliunde, whether oral or written, which is intended or tends to vary or contradict a complete and enforceable agreement embodied in a document. Why is there such a rule? When an agreement has been reduced into writing, they are presumed to have intended the writing as the ONLY EVIDENCE of their agreement, and, therefore, they are supposed to have embodied all the terms of such agreement. Consequently, all prior or contemporaneous collateral stipulations which the parties might have had and which do not appear in writing, are presumed to have been waived or abandoned by them, and therefore, not provable.

Availability of the Original Document

What the rule prohibits

What kind of documents does the rule apply?

Parol Evidence Rule Presupposes that the original document is available in court.

Prohibits the varying of terms of a written agreement.

Applies only to documents which are contractual in nature, including wills.

Best Evidence Rule Contemplates the situation wherein the original writing is not available and/or there is a dispute as to whether said writing is the original. Prohibits the introduction of substitutionary evidence in lieu of the original document regardless of whether or not it varies the contents of the original. Applies to all kinds of writings.

Layno. Rabajante. Morana. Reyes. Suyat. Baraoidan|

14

[JARA POWER NOTES ON EVIDENCE]

2011-2012

Who may invoke?

Only when the controversy is between the parties to the written agreement, their privies, or any party directly affected thereby.

Can be invoked by any party to an action regardless of whether or not such party has participated in the writing involved.

notarized. The substantive rule in the Civil Code provides that: ―No will shall pass either real or personal estate unless it is proved and allowed in the probate court.‖ Therefore, is Sec. 19, Rule 132 applicable to wills? Hindi phowZ. Jejeje.

If the last paragraph in Section 9 of Rule 130 was not included, would parol evidence rule still be applicable? Yes. The Civil Code is pretty clear on this one: Art. 789. When there is an imperfect description, or when no person or property exactly answers the description, mistakes and omissions must be corrected, if the error appears from the context of the will or from extrinsic evidence, excluding the oral declarations of the testator as to his intention; and when an uncertainty arises upon the face of the will, as to the application of any of its provisions, the testator's intention is to be ascertained from the words of the will, taking into consideration the circumstances under which it was made, excluding such oral declarations. AUTHENTICATION AND PROOF OF DOCUMENTS Let us discuss the process of authenticating a document. What is a public document? What is a private document?

Distinguish a notarized deed of sale from a notarized will. A notarized deed of sale is a public document and needs no authentication. A notarized will, by express provision of the Civil Code and the Rules of Court, needs to undergo authentication. Even if such is notarized, it cannot be considered a public document. Why is there a need to authenticate a notarized will? Public policy requires it. Courts must at all times give respect to testamentary intent. In order to prevent fraud, authentication of wills is mandatory. How may the genuineness of a notarized deed be assailed? It may be assailed by discrediting the genuineness of the signature, or the handwriting, or the identity of the public officer who prepared the same (not sure. But sounds right).

Public Documents are: How do you prove a private document? 1. The written official acts, or records of the official acts of the sovereign authority, official bodies and tribunals, and public officers, whether of the Philippines, or of a foreign country; 2. Documents acknowledged before a notary public except last wills and testaments; and 3. Public records, kept in the Philippines, or private documents required by law to be entered therein.

The due execution and authenticity of a private document must be proved either by: 1. Anyone who saw the document executed or written; 2. Evidence of the genuineness of the signature or handwriting of the maker. It is therefore NOT NECESSARY that the witness be an eyewitness Must all private documents undergo the process of authentication?

All other documents are private. Is a notarized will a public document? NO. Last wills and testament must undergo an authentication process even if they are

NO. The following are private writings which may be admitted in evidence without previous proof of its authenticity and due execution:

Layno. Rabajante. Morana. Reyes. Suyat. Baraoidan|

15

2011-2012

[JARA POWER NOTES ON EVIDENCE]

1. When the genuineness and due execution of the document is admitted by the adverse party; 2. When such genuineness and due execution are immaterial to the issue; 3. When the document is an ANCIENT DOCUMENT. 4. Rule on Actionable Documents – when the adverse party fails to specifically deny under oath the genuineness and due execution of the actionable document and to set forth what he claims to be the facts, the same shall be deemed admitted (Rule 8,Sec.8). 5. Request for Admission – if the party fails to specifically deny the matters on which the admission is requested within the period given, the genuineness and due execution shall be deemed admitted (Rule 26). 6. Pre-Trial – the parties can enter stipulations and admit the genuineness and due execution. What are ancient documents? What is the rule with respect to such? Requirements for Ancient Document Rule: 1. The private document is more than 30 years old; 2. It is produced from custody in which it would naturally be found if genuine; 3. It is unblemished by any alterations or circumstances of suspicions. It is important to note, however, that the ancient document rule applies only when there are no available witnesses to testify as to the authenticity of the document. A documentary evidence in an unofficial language was presented to the court. How can a proponent convince the court that the translation is credible? Documents written in an unofficial language shall not be admitted as evidence, unless accompanied with a translation into English or Filipino. To avoid interruption of proceedings, parties or their attorneys are directed to have such translation prepared before trial. To convince the court, present the evidence, get a translation, have a translator take the witness stand as an expert witness, qualify the witness,

make him testify and read the whole thing, then let him translate the language. How do you impeach judicial records? 1. By evidence of want of jurisdiction of the court or judicial officer; 2. Collusion between the parties; 3. Extrinsic fraud on the part of the party offering the record, in respect of the proceedings. What happened in the case of US vs. Gregorio? In the suit instituted for the payment of a certain sum of money, judgment was rendered wherein the debtor was sentenced to pay to the plaintiff P275.92, with interest thereon, and the costs. For the execution of the said judgment, two rural properties belonging to the debtor were attached. Bernardo Gregorio requested the deputy sheriff to exclude the said realty from the attachment, alleging that he was the owner of the land situated in Tambogon, one of the properties levied upon for the reason that he had acquired it by purchase from the judgment debtor, Balistoy, in 1905, prior to the filing of the complaint. In order that the claim of intervention presented to the sheriff might prosper, Bernardo Gregorio attached thereto the document Exhibit D, at the end of which and among other particulars appears the memorandum dated in Libog as of February 22, 1905, and signed by Eustaquio Balistoy, Lorenzo Gregorio, and Cirilo Valla, and in which Balistoy states that he bought the land referred to in the said document from Luis Balistoy and sold it to Bernardo Gregorio for P300, wherefore he signed as such vendor. A complaint was filed in each of the two aforesaid causes in the Court of First Instance of Albay, charging each of the defendants with the crime of the falsification of a private document. Issue: WON Balistoy, with intent to injure his creditor, Pedro Salazar, and for the purpose of avoiding the attachment and sale of one of the properties did execute or write the said memorandum whereby, on February 25, 1905, he made or simulated a conveyance of one of the attached properties in favor of the said Bernardo Gregorio, according to the aforesaid copy, when in fact the said memorandum was written in April, 1908.

Layno. Rabajante. Morana. Reyes. Suyat. Baraoidan|

16

2011-2012

[JARA POWER NOTES ON EVIDENCE]

Held: In the charge filed in this cause against the vendor and the vendee of the land in question, it is stated that these parties, the defendants, simulated the said memorandum of sale or conveyance of the land with the intent to injure the creditor, Pedro Salazar; but as the original document, setting forth the said memorandum, was not presented, but merely a copy thereof, and furthermore, as it could not be ascertained who had the original of the document containing the memorandum in question, nor the exact date when the latter was written; the said memorandum, presumed to be simulated and false, was not literally compared by the sheriff who testified that he had seen its original for but a few moments, nor by any officer authorized by law to certify to documents and proceedings such as are recorded in notarial instruments, nor even by two witnesses who might afterwards have been able to testify before the court that the copy exhibited was in exact agreement with its original; therefore, on account of these deficiencies, doubt arises as to whether the original of the document, Exhibit D, really existed at all, and whether the memorandum at the foot of the said exhibit is an exact copy of that alleged to have been written at the end of the said original document. In criminal proceedings for the falsification of a document, it is indispensable that the judges and courts have before them the document alleged to have been simulated, counterfeited, or falsified, in order that they may find, pursuant to the evidence produced in the cause, whether or not the crime of falsification was committed, and also, at the same time, to enable them to determine the degree of each defendant's liability in the falsification under prosecution. Through the lack of the original document containing the memorandum alleged to be false, it is improper to hold, with only a copy of the said original in view, that the crime prosecuted was committed; and although, judging from the testimony of the witnesses who were examined in the two consolidated causes, there is reason to entertain much doubt as to the defendants' innocence, yet, withal, this case does not furnish decisive and conclusive proof of their respective guilt as co-principals of the crime charged. Defendants in a criminal cause are always presumed to be innocent until their guilt be fully proven, and, in case of reasonable doubt and when their guilt is not satisfactorily shown, they are entitled to a judgment of acquittal.

What happened in Government vs. Martinez? In the cadastral proceedings instituted in the Court of Land Registration for the settlement of titles to lands in the municipality of Iloilo, Province of Iloilo, Dolores and Carmen Martinez appeared claiming to be the owners of lots Nos. 873 and 450. They alleged that they were in possession thereof for about twenty-five years, having acquired them by donation from Maria Sarlabus, and that their predecessors in interest had possession of the same for at least three years prior to said donation. On the otherhand, Julio Salvador, through his attorney, entered his appearance and claimed title to said lots, alleging that he was in actual possession thereof, and that his predecessors in interest had been in possession before him for at least fourteen years. To support their claim, they presented before the court the testimony of two witnesses and a certified copy issued by the acting registrar of deeds of Iloilo dated May 13, 1914. The CFI of Iloilo ruled in favor of Julio Salvador on the ground that, in the opinion of the court, it was proved that the Martinez sisters had sold said land to one named Domenech and that the latter, in return, sold it to Julio Salvador, who could, therefore, be considered owner of the disputed lots. Dolores and Carmen Martinez excepted to said judgment and filed a bill of exceptions after their motion for new trial had been overruled, and they had excepted to the order overruling said motion. Issue: WON the trial court erred in admitting the copy of the record of a supposed document of sale presented by Julio Salvador, in support of his claim of title without the disappearance or loss of the original document having been previously proved. Held: Yes. Section 299 of the Code of Civil Procedure provides: "The written acts of record of the acts of the sovereign authority, of official bodies and tribunals and of public officers, legislative, judicial, and executive of the Philippine Islands, or of the United States, or of any States of the United States or of a foreign country, and public records kept in the Philippine Islands of private writings are public writings. A copy of a public writing, duly certified to be a

Layno. Rabajante. Morana. Reyes. Suyat. Baraoidan|

17

2011-2012

[JARA POWER NOTES ON EVIDENCE]

true copy thereof, is admissible evidence in like cases and in like effect as the original writing." The certified copy issued by the acting registrar of deeds of Iloilo, Exhibit 2, is not a true copy of the document of sale which is said to have been executed by the Martinez sisters in favor of Antonio Domenech, but of the recital appearing in the books of said registry with respect to the urban property, consisting of those two lots. In the case at bar, Julio Salvador, had not first complied with the provisions of section 321 of the Code of Civil Procedure. The Supreme Court ruled that the best obtainable evidence should be adduced to prove every disputed fact, and a failure to produce it, but an attempt instead to sustain the issue by inferior evidence, will authorize the inference that the party does not furnish the best evidence because it would tend to defeat, instead of sustaining, the issue of his part. In requiring the production of the best evidence applicable to each particular fact, it is meant that no evidence shall be received which is merely substitutionary in its nature, so long as the original evidence can be had. Undoubtedly the best evidence of the contents of a written instrument consists in the actual production of the instrument itself, and the general rule is that secondary evidence of its contents cannot be admitted until the nonproduction of the original has been satisfactory accounted for. Secondary evidence of the contents of writings if admitted on the theory that the original cannot be produced by the party by whom the evidence is offered, within a reasonable time by the exercise of reasonable diligence. And ordinarily secondary evidence is not admissible until the nonproduction of the primary evidence has been sufficiently accounted for. The contents of a lost instrument cannot be proved unless it appears that reasonable search has been made in the place where the paper was last know to have been, and if not found there, that inquiry has been made of the person last known to have had its custody.

of the party interested, and in the place where it is most likely to be found. . . . But to justify admission of secondary evidence of a deed, it is not necessary to prove its loss beyond all possibility of mistake. A reasonable probability of its loss is sufficient; and this may be shown by a bona fide and diligent search, fruitless made for it in places where it is likely to be found. As the failure of the oppositor Salvador to present the original document in question was not accounted for; as it is not proper to suppose that the original could not have been presented within a reasonable time if he had exercised due diligence for he or his counsel had the means, opportunity and time to find the original if it really existed; as no proof was adduced that said document had been lost, or destroyed, or that proper search therefor was made in the general files of notarial documents in the City of Manila, or that an attempt was made to secure a copy thereof if it existed in said files; as the notary, Gregorio Yulo, a person well known in Iloilo, was not asked directly and clearly as to the whereabouts of said document or some particular or data it in order to obtain from him some conclusive and categorical answer; as said notary has not been presented at the trial to be examined on these points; and, lastly, as it was not shown that the party interested in the presentation of said document who is Julio Salvador, had made a diligent and proper, but fruitless, search for said document in any place where it could probably be found — therefore the secondary evidence presented by the oppositor, consisting of the testimony of the witnesses, Saez and Madrenas, and the certified copy issued by the registrar of deeds of Iloilo, Exhibit 2, is of no value for the purpose intended and such evidence was improperly considered by the court in reaching the conclusion that said Julio Salvador was the owner of the lots in question. The judgment appealed from is therefore reversed and the claims of Julio Salvador is denied; and we declare that the two lots Nos. 873 and 450 should be adjudicated to the appellants Carmen and Dolores Martinez and be registered in their name. No special pronouncement is made as to costs. So ordered.

In accordance with the rule set forth in the next preceding paragraph parol evidence of the contents of a will is inadmissible, unless it is first shown that diligent and unavailing search has been made for the original, by or at the request

Layno. Rabajante. Morana. Reyes. Suyat. Baraoidan|

18

2011-2012

[JARA POWER NOTES ON EVIDENCE]

What happened in the case of MCC vs. Ssangyong? Facts: Petitioner MCC Industrial Sales (MCC), a domestic corporation with office at Binondo, Manila, is engaged in the business of importing and wholesaling stainless steel products. One of its suppliers is the Ssangyong Corporation (Ssangyong), an international trading company with head office in Seoul, South Korea and regional headquarters in Makati City, Philippines. The two corporations conducted business through telephone calls and facsimile or telecopy transmissions. Ssangyong would send the pro forma invoices containing the details of the steel product order to MCC; if the latter conforms thereto, its representative affixes his signature on the faxed copy and sends it back to Ssangyong, again by fax. Due to the failure of MCC to put up a Letter of Credit, Ssangyong sued MCC in the RTC. After Ssangyong rested its case, defendants filed a demurrer to evidence, alleging that Ssangyong failed to present the original copies of the pro forma invoices on which the civil action was based. In an Order dated April 24, 2003, the court denied the demurrer, ruling that the documentary evidence presented had already been admitted in the December 16, 2002 Order and their admissibility finds support in RA 8792, otherwise known as the Electronic Commerce Act of 2000. Considering that both testimonial and documentary evidence tended to substantiate the material allegations in the complaint, Ssangyong's evidence sufficed for purposes of a prima facie case. RTC ruled in favor of Ssangyong, so did the CA. Issue: WON the printout and/or photocopies of facsimile transmissions are electronic evidence and admissible as such. Held: Although the parties did not raise the question whether the original facsimile transmissions are "electronic data messages" or "electronic documents" within the context of the Electronic Commerce Act (the petitioner merely assails as inadmissible evidence the photocopies of the said facsimile transmissions), we deem it appropriate to determine first whether the said fax transmissions are indeed within the coverage of RA 8792 before ruling on whether the photocopies thereof are covered by the law.

RA 8792, otherwise known as the Electronic Commerce Act of 2000, considers an electronic data message or an electronic document as the functional equivalent of a written document for evidentiary purposes. The Rules on Electronic Evidence regards an electronic document as admissible in evidence if it complies with the rules on admissibility prescribed by the Rules of Court and related laws and is authenticated in the manner prescribed by the said Rules. An electronic document is also the equivalent of an original document under the Best Evidence Rule, if it is a printout or output readable by sight or other means, shown to reflect the data accurately. Thus, to be admissible in evidence as an electronic data message or to be considered as the functional equivalent of an original document under the Best Evidence Rule, the writing must foremost be an "electronic data message" or an "electronic document." According to the deliberations in Congress, when Congress formulated the term "electronic data message," it intended the same meaning as the term "electronic record" in the Canada law. This construction of the term "electronic data message," which excludes telexes or faxes, except computer-generated faxes, is in harmony with the Electronic Commerce Law's focus on "paperless" communications and the "functional equivalent approach" that it espouses. In fact, the deliberations of the legislature are replete with discussions on paperless and digital transactions. Facsimile transmissions are not, in this sense, "paperless," but verily are paperbased. Accordingly, in an ordinary facsimile transmission, there exists an original paperbased information or data that is scanned, sent through a phone line, and reprinted at the receiving end. Be it noted that in enacting the Electronic Commerce Act of 2000, Congress intended virtual or paperless writings to be the functional equivalent and to have the same legal function as paper-based documents. Further, in a virtual or paperless environment, technically, there is no original copy to speak of, as all direct printouts of the virtual reality are the same, in all respects, and are considered as originals. Ineluctably, the law's definition of "electronic data message," which, as aforesaid, is interchangeable with "electronic document," could not have included facsimile transmissions, which have an original paper-based copy as sent and a paper-based facsimile copy as

Layno. Rabajante. Morana. Reyes. Suyat. Baraoidan|

19

2011-2012

[JARA POWER NOTES ON EVIDENCE]

received. These two copies are distinct from each other and have different legal effects. While Congress anticipated future developments in communications and computer technology when it drafted the law, it excluded the early forms of technology, like telegraph, telex and telecopy (except computer-generated faxes, which is a newer development as compared with the ordinary fax machine to fax machine transmission), when it defined the term "electronic data message." We, therefore, conclude that the terms "electronic data message" and "electronic document," as defined under the Electronic Commerce Act of 2000, do not include a facsimile transmission. Accordingly, a facsimile transmission cannot be considered as electronic evidence. It is not the functional equivalent of an original under the Best Evidence Rule and is not admissible as electronic evidence. Since a facsimile transmission is not an "electronic data message" or an "electronic document," and cannot be considered as electronic evidence by the Court, with greater reason is a photocopy of such a fax transmission not electronic evidence. Ang vs. CA? Facts: Rustan Ang and private respondent Irish Sagud were sweethearts on an ―on-and-off‖ basis towards the end of 2004. When Irish learned afterwards that Rustan had taken a livein partner (now his wife), whom he had gotten pregnant, Irish broke up with him. Before Rustan got married, however, he got in touch with Irish and tried to convince her to elope with him, saying that he did not love the woman he was about to marry. Irish rejected the proposal and told Rustan to take on his responsibility to the other woman and their child. One day, Irish received through multimedia message service a picture of a naked woman with spread legs with her face superimposed on the figure. The message allegedly came from Rustan‘s number. After she got the obscene picture, Irish got other text messages from Rustan. He boasted that it would be easy for him to create similarly scandalous pictures of her. And he threatened to spread the picture he sent through the internet. Subsequently, a criminal complaint for violation against women through harassment was filed against Rustan. During trial, Rustan claimed that Irish merely sought his help to

identify a prankster who was sending her malicious text messages. Rustan got the sender‘s number and, pretending to be Irish, contacted the person. Rustan claims that he got back obscene messages from the prankster, which he forwarded to Irish from his cellphone. This explained, he said, why the obscene messages appeared to have originated from his cellphone number. After trial, the RTC gave more credence to Irish‘s testimony. The lower court found Irish‘s testimony completely credible, given in an honest and spontaneous manner, and hence convicted Rustan of the crime of violence against women through harassment. Upon appeal, CA affirmed the RTC decision. Rustan interposed, among others, that the obscene picture should be inadmissible as evidence since such picture sent to Irish through a text message constitutes an electronic document and thus should be authenticated under the Rules on Electronic Evidence. Issue: WON the RTC properly admitted in evidence the obscene picture presented in the case. Held: Yes. The Court noted that the objection of Rustan as to the admissibility of the obscene picture as evidence was already too late since he should have objected to the admission of the picture on such ground at the time it was offered in evidence. He should be deemed to have already waived such ground for objection. Besides, the Rules on Electronic Evidence do not apply to the present criminal action. It was held that it applies only to civil actions, quasijudicial proceedings, and administrative proceedings. Accordingly, the Court affirmed RTC‘s admission in evidence of the subject obscene picture. ELECTRONIC EVIDENCE Note: Codal provisions lang ang labanan dito. Please read the rules. When is electronic evidence regarded as being the equivalent of an original document under the Best Evidence Rule? An electronic document shall be regarded as the equivalent of an original document under the Best Evidence Rule if it is a printout or output readable by sight or other means, shown to reflect the data accurately.

Layno. Rabajante. Morana. Reyes. Suyat. Baraoidan|

20

2011-2012

[JARA POWER NOTES ON EVIDENCE]

May a print-out or photocopy or both of facsimile transmissions be considered as electronic evidence and admissible as the term is defined under the E-Commerce Act and its implementing rules? NO. The definitions under the E-Commerce Act, its IRR and the Rules on Electronic Evidence, at first glance, convey the impression that facsimile transmissions are electronic data messages or electronic documents because they are sent by electronic means. When the Senate voted to adopt the term ―electronic data message,‖ it was consonant with the explanation of Senator Miriam Defensor-Santiago that it would not apply ―to telexes or faxes, except computer generated faxes, unlike the UN model law on electronic commerce.‖ There is no question that when Congress formulated the term ―electronic data message,‖ it intended the same meaning as the term ―electronic record‖ in the Canada law. This construction of the term ―electronic data message,‖ which excludes telexes or faxes, except computer generated faxes, is in harmony with the Electronic Commerce Law‘s focus on ―paperless‖ communications and the ―functional equivalent approach‖ that it espouses. In fact, the deliberations of the Legislature are replete with discussions on paperless and digital transactions. Since a facsimile transmission is not an ―electronic data message‖ or an ―electronic document,‖ and cannot be considered as electronic evidence by the Court, with greater reason is a photocopy of such a fax transmission not considered electronic evidence (MCC Industrial Sales, Corp. v. Ssangyong Corp., G.R. No. 170633). How does one treat “text messages” as evidence? Text messages have been classified as ―ephemeral electronic communication‖ under Sec. 1 (k), Rule 2 of the Rules on Electronic Evidence, and ―shall be proven by the testimony of a person who was a party to the same or has personal knowledge thereof.‖ ―Ephemeral electronic communication‖ refers to telephone conversations, text messages and other electronic forms of communication the evidence of which is not recorded or retained (VidallonMagtolis v. Salud., A.M. No. CA-05-20-P).

TESTIMONIAL EVIDENCE QUALIFICATIONS AND DISQUALIFICATIONS OF WITNESSES What are the necessary qualifications in order for one to be a witness? All persons who can perceive, and perceiving, can make known their perception to others, may be witnesses. Are there any disqualifications? Disqualifications of a witness: 1. Disqualification by reason of mental incapacity or immaturity; 2. Disqualification by reason of marriage; 3. Dead Man‘s statute; 4. Privileged communications; 5. Parental and filial privileges; 6. Art. 821 of the New Civil Code disqualifies those who have been convicted of falsification of a document, perjury or false testimony from being witnesses to a will. 7. Section 17, Rule 119 of the Rules of Court requires that the accused sought to be discharged to be state witness has not at any time been convicted of any offense involving moral turpitude. The same requirement is provided for a state witness under R.A. 6981 or Witness Protection, Security and Benefit Act. Are deaf-mutes disqualified to testify as witnesses? NO. deaf-mutes are not necessarily incompetent as witnesses. They are competent where they: (a) can understand and appreciate the sanctity of an oath; (b) can comprehend the facts they are going to testify to; and (c) can communicate their ideas through a qualified interpreter (People v. Tuangco, 345 SCRA 429) Distinguish competency of a witness and credibility of a witness. Competency of a witness Has reference to the qualifications of a witness as his capacity to perceive and his capacity

Credibility of a witness Refers to the believability of the witness and has nothing to do with the law or the rules. it refers

Layno. Rabajante. Morana. Reyes. Suyat. Baraoidan|

21

2011-2012 to communicate his perception to others. It also includes the absence of any disqualifications imposed upon a witness.

[JARA POWER NOTES ON EVIDENCE]

to the weight and the trustworthiness or reliability of the testimony.

When do you determine the qualifications of a witness? The qualifications and disqualifications of a witnesses are determined as of the time said witnesses are produced for examination in court or at the taking of their depositions. With respect to children of tender years, their competence at the time of the occurrence to be testified should also be taken into account, especially if such event took place long before their production as witnesses. Distinguish spousal immunity from marital privilege. Disqualification by Reason of Marriage

Period for Prohibition Scope of Prohibition

Exception

Applies only during their marriage Any communication received by one from the other

1. 2.

Disqualification by Reason of Privileged Communication between spouses Applies even after the marriage

As to communication received in confidence by one from the other during their marriage Unless there is consent by the other spouse; or Except in a civil case by one against the other, or in a criminal case fro a crime committed by one against the other or the latter‘s direct descendants or ascendants.

Can you compel an adverse party to be your witness? Yes, in civil cases. Such party is then treated as a hostile witness. However, there must be compliance with the requirements of Rule 25 and Rule 26 of the Rules of Court (SHOTGUN QUESTION).

RULE ON THE EXAMINATION OF A CHILD WITNESS When is the rule on examination of child witnesses applicable? Examination of child witnesses who are victims, accused, and witnesses to crime. It shall apply in all criminal proceedings and non-criminal proceedings involving child witnesses How shall the rule be construed? Liberally, to uphold the best interests of the child, and to promote maximum accommodation of child witnesses, without prejudice to the rights of the accused. Who may be considered a child witness? Any person who, at the time of giving testimony, is below the age of 18 years. In child abuse cases, a child includes one over 18 years but is found by the court as unable to fully take care of himself or protect himself from abuse, neglect, cruelty, exploitation, or discrimination because of a physical or mental disability or condition. Who is a “guardian ad litem”? A person appointed by the court where the case is pending for a child who is a victim of, accused of, or a witness to a crime to protect the best interests of the said child. What are the powers and functions of the Guardian Ad Litem? Section 5. Guardian ad litem. – (a) The court may appoint a guardian ad litem for a child who is a victim of, accused of, or a witness to a crime to promote the best interests of the child. In making the appointment, the court shall consider the background of the guardian ad litem and his familiarity with the judicial process, social service programs, and child development, giving preference to the parents of the child, if qualified. The guardian ad litem may be a member of the Philippine Bar. A person who is a witness in any proceeding involving the child cannot be appointed as a guardian ad litem. (b) The guardian ad litem: (1) Shall attend all interviews, depositions, hearings, and trial

Layno. Rabajante. Morana. Reyes. Suyat. Baraoidan|

22

2011-2012

[JARA POWER NOTES ON EVIDENCE]

proceedings in which a child participates; (2) Shall make recommendations to the court concerning the welfare of the child; (3) Shall have access to all reports, evaluations, and records necessary to effectively advocate for the child, except privileged communications; (4) Shall marshal and coordinate the delivery of resources and special services to the child; (5) Shall explain, in language understandable to the child, all legal proceedings, including police investigations, in which the child is involved; (6) Shall assist the child and his family in coping with the emotional effects of crime and subsequent criminal or non-criminal proceedings in which the child is involved; (7) May remain with the child while the child waits to testify; (8) May interview witnesses; and (9) May request additional examinations by medical or mental health professionals if there is a compelling need therefor. (c) The guardian ad litem shall be notified of all proceedings but shall not participate in the trial. However, he may file motions pursuant to sections 9, 10, 25, 26, 27 and 31(c). If the guardian ad litem is a lawyer, he may object during trial that questions asked of the child are not appropriate to his developmental level. (d) The guardian ad litem may communicate concerns regarding the child to the court through an officer of the court designated for that purpose. (e) The guardian ad litem shall not testify in any proceeding concerning any information, statement, or opinion received from the child in the course of serving as a guardian ad litem, unless the court finds it necessary to promote the best interests of the child. (f) The guardian ad litem shall be presumed to have acted in good faith in compliance with his duties described in sub-section (b).

What is the rule with regard to qualification of the child as a witness?

the

Every child is presumed qualified to be a witness. However, the court shall conduct a competency examination of a child, motu proprio or on motion of a party, when it finds that substantial doubt exists regarding the ability of the child to perceive, remember, communicate, distinguish truth from falsehood, or appreciate the duty to tell the truth in court. Who must prove the need for competency examinations? A party seeking a competency examination must present proof of necessity of competency examination. The age of the child by itself is not a sufficient basis for a competency examination. Moreover, to rebut the presumption of competence enjoyed by a child, the burden of proof lies on the party challenging his competence. Is the competency examination a final determination as to the competence of the child? No. The court has the duty of continuously assessing the competence of the child throughout his testimony. When may the court appoint an interpreter for the child? Section 9. Interpreter for child. – (a) When a child does not understand the English or Filipino language or is unable to communicate in said languages due to his developmental level, fear, shyness, disability, or other similar reason, an interpreter whom the child can understand and who understands the child may be appointed by the court, motu proprio or upon motion, to interpret for the child.

What are “the best interests of the child”? The totality of the circumstances and conditions as are most congenial to the survival, protection, and feelings of security of the child and most encouraging to his physical, psychological, and emotional development. It also means the least detrimental available alternative for safeguarding the growth and development of the child.

(b) If a witness or member of the family of the child is the only person who can serve as an interpreter for the child, he shall not be disqualified and may serve as the interpreter of the child. The interpreter, however, who is also a witness, shall testify ahead of the child (c) An interpreter shall take an oath or affirmation to make a true and accurate interpretation.

Layno. Rabajante. Morana. Reyes. Suyat. Baraoidan|

23

2011-2012

[JARA POWER NOTES ON EVIDENCE]

When may the court appoint a facilitator to ask questions to the child? Section 10. Facilitator to pose questions to child. (a) The court may, motu proprio or upon motion, appoint a facilitator if it determines that the child is unable to understand or respond to questions asked. The facilitator may be a child psychologist, psychiatrist, social worker, guidance counselor, teacher, religious leader, parent, or relative. (b) If the court appoints a facilitator, the respective counsels for the parties shall pose questions to the child only through the facilitator. The questions shall either be in the words used by counsel or, if the child is not likely to understand the same, in words that are comprehensible to the child and which convey the meaning intended by counsel. (c) The facilitator shall take an oath or affirmation to pose questions to the child according to the meaning intended by counsel. Who is a “support person”? Person chosen by the child to accompany him to testify at or attend a judicial proceeding or disposition to provide emotional support for him. What are the rules relevant to support persons? Section 11. Support persons. – (a) A child testifying at a judicial proceeding or making a deposition shall have the right to be accompanied by one or two persons of his own choosing to provide him emotional support. (1) Both support persons shall remain within the view of the child during his testimony. (2) One of the support persons may accompany the child to the witness stand, provided the support person does not completely obscure the child from the view of the opposing party, judge, or hearing officer. (3) The court may allow the support person to hold the hand of the child or take other appropriate steps to provide emotional support to the child in the course of the proceedings. (4) The court shall instruct the support persons not to prompt, sway, or influence the child during his testimony.

(b) If the support person chosen by the child is also a witness, the court may disapprove the choice if it is sufficiently established that the attendance of the support person during the testimony of the child would pose a substantial risk of influencing or affecting the content of the testimony of the child. May the courtroom environment be adjusted to make the child more comfortable? Yes. Section 13 says: ―To create a more comfortable environment for the child, the court may, in its discretion, direct and supervise the location, movement and deportment of all persons in the courtroom including the parties, their counsel, child, witnesses, support persons, guardian ad litem, facilitator, and court personnel. The child may be allowed to testify from a place other than the witness chair. The witness chair or other place from which the child testifies may be turned to facilitate his testimony but the opposing party and his counsel must have a frontal or profile view of the child during the testimony of the child. The witness chair or other place from which the child testifies may also be rearranged to allow the child to see the opposing party and his counsel, if he chooses to look at them, without turning his body or leaving the witness stand. The judge need not wear his judicial robe. Nothing in this section or any other provision of law, except official incour identification provisions, shall be construed to require a child to look at the accused. Accommodations for the child under this section need not be supported by a finding of trauma to the child.‖ May the child use “testimonial aids” during his / her testimony? Yes. The court shall permit anatomically-correct dolls, mannequins, or any demonstrative device to testimony.

a child to use dolls, puppets, drawings, other appropriate assist him in his

May the child be allowed to have an “emotional security item”? Yes. While testifying, a child shall be allowed to have an item of his own choosing such as a blanket, toy, or doll.

Layno. Rabajante. Morana. Reyes. Suyat. Baraoidan|

24

2011-2012

[JARA POWER NOTES ON EVIDENCE]

Does the examination of the child need to follow strictly the usual procedure for examination of witnesses? No. Some exceptions are: 1. The court may allow leading questions in all stages of examination of a child if it will further the interests of justice 2. The court may allow the child witness to testify in a narrative form 3. Objections must be couched in a manner so as not to mislead, confuse, frighten, or intimidate the child. Does the child witness’ testimony need to be corroborated? No. His / her testimony, if credible by itself, shall be sufficient. On what grounds may the court exclude the public during the examination of a child witness? Such an order may be made to protect the right to privacy of the child or if the court determines on the record that requiring the child to testify in open court would cause psychological harm to him, hinder the ascertainment of truth, or result in his inability to effectively communicate due to embarrassment, fear, or timidity. Does the child need to testify inside the courtroom? No. The prosecutor, counsel or the guardian ad litem may apply for an order that the testimony of the child be taken in a room outside the courtroom and be televised to the courtroom by live-link television. Also, the judge may question the child in chambers, or in some comfortable place other than the courtroom, in the presence of the support person, guardian ad litem, prosecutor, and counsel for the parties. The questions of the judge shall not be related to the issues at trial but to the feelings of the child about testifying in the courtroom. What are the grounds to allow testimony by live-link television? If there is a substantial likelihood that the child would suffer trauma from testifying in the presence of the accused, his counsel or the prosecutor as the case may be.

What other similar measures may the court take aside from live-link television? The prosecutor or the guardian ad litem may apply for an order that the chair of the child or that a screen or other device be placed in the courtroom in such a manner that the child cannot see the accused while testifying. If the court grants an application to shield the child from the accused while testifying in the courtroom, the courtroom shall be arranged to enable the accused to view the child. May the testimony of the child be given through videotaped deposition? Yes. If the court finds that the child will not be able to testify in open court at trial, it shall issue an order that the deposition of the child be taken and preserved by videotape. Does the hearsay rule apply to children’s testimony? Not strictly, in child abuse cases. A statement made by a child describing any act or attempted act of child abuse, not otherwise admissible under the hearsay rule, may be admitted in evidence in any criminal or non-criminal proceeding subject to certain rules found in Section 28 (a). May the court admit videotape and audiotape interviews as evidence? Yes, under certain conditions imposed under Section 29. What is the Sexual Abuse Shield Rule? Section 30. Sexual abuse shield rule. (a) Inadmissible evidence. - The following evidence is not admissible in any criminal proceeding involving alleged child sexual abuse: (1) Evidence offered to prove that the alleged victim engaged in other sexual behavior; and (2) Evidence offered to prove the sexual predisposition of the alleged victim. (b) Exception. - Evidence of specific instances of sexual behavior by the alleged victim to prove that a person other than the accused was the source of semen, injury, or other physical evidence shall be admissible. A party intending to offer such evidence must: (1) File a written

Layno. Rabajante. Morana. Reyes. Suyat. Baraoidan|

25

2011-2012

[JARA POWER NOTES ON EVIDENCE]

motion at least fifteen (15) days before trial, specifically describing the evidence and stating the purpose for which it is offered, unless the court, for good cause, requires a different time for filing or permits filing during trial; and (2) Serve the motion on all parties and the guardian ad litem at least three (3) days before the hearing of the motion. Before admitting such evidence, the court must conduct a hearing in chambers and afford the child, his guardian ad litem, the parties, and their counsel a right to attend and be heard. The motion and the record of the hearing must be sealed and remain under seal and protected by a protective order set forth in section 31(b). The child shall not be required to testify at the hearing in chambers except with his consent. How is the privacy of the child protected? Any records regarding the child shall be confidential and kept under seal. Except upon written request and order of the court, the record may only be released to certain persons under Section 31(a). Moreover, any videotape or audiotape of a child that is part of the court record shall be under a protective order as provided by Section 31(b). The court may issue additional protection orders to protect the child‘s privacy. Whoever publishes or causes to be published the identifying information of the child or the immediate family of the child shall be liable to the contempt power of the court. Any videotape or audiotape of a child made part of the court record shall be destroyed after 5 years from the date of entry of judgment. How are the ordinary rules of Court applied? Suppletorily. When do you challenge the qualification of the witness? Before he is made to testify on the matters which he/she was presented. Failure to challenge will uphold the qualification of the witness. Kaya nga daw PRELIMINARY EXAMINATION ang tawag sabi ni Jara.

Distinguish the manner of taking the testimony of a child witness from that of an ordinary witness. Child Witness Only the judge is allowed to ask questions to a child witness during preliminary examination. Leading questions are allowed. Testimony in a narrative form is allowed. The child witness is assisted by a facilitator.

Ordinary Witness Opposing counsels are allowed to ask questions during preliminary examination. Leading questions are generally not allowed. Testimony in a narrative form is not allowed. An ordinary witness is not assisted by a facilitator.

What is the Dead Man’s Statute? Parties or assignors of parties to a case, or persons in whose behalf a case is prosecuted, against an executor or administrator or other representative of a deceased person, or against a person of unsound mind, upon a claim or demand against the estate of such deceased person or against such person of unsound mind, cannot testify as to any matter of fact occurring before the death of such deceased person or before such person became of unsound mind. (NOTE: CODAL. VERBATIM. REQUIRED.) What are the elements of the “Dead Man’s Statute?” 1. The defendant in the case is the executor or administrator or a representative of the deceased or the person of unsound mind; 2. The suit is upon a claim by the plaintiff against the estate of said deceased person or person of unsound mind; 3. The witness is the plaintiff, or an assignor of that party, or a person in whose behalf the case is prosecuted; 4. The subject of the testimony is as to any matter of fact occurring before the death of such person or before such person became of unsound mind.

Layno. Rabajante. Morana. Reyes. Suyat. Baraoidan|

26

2011-2012

[JARA POWER NOTES ON EVIDENCE]

A creditor filed a case against the debtor. In case the creditor dies and the debtor is alive and of sound mind, does the rule apply? No. The first element of the Dead Man‘s Statute is absent in this case. The person who must have been disabled or who must have died in this case is the defendant. The defendant must be the one therefore being defended in this case. Under what instances does the rule not apply? With respect to the FIRST ELEMENT: 1. Witnesses who are neither parties the case, their assignors, nor persons in whose behalf the case is prosecuted; 2. A counterclaim has been interposed by the defendant as the plaintiff would thereby be testifying in his defense. With respect to the SECOND ELEMENT: 1. Defendant who is sued in his individual, rather in a representative capacity.

What happened in the case of Icard v. Masigan? Facts: For services rendered in connection with the development and location of certain mining claims, Joseph K. Icard filed a claim of P2,000 against the estate of his deceased father George M. Icard. The claim having been allowed by the commissioner on claims, the administrator appealed to the Court of First Instance, where it was likewise allowed. DOCTRINE: the Dead Man‘s Statute is designed to close the lips of the party plaintiff when death has closed the lips of the party defendant, in order to remove from the surviving party the temptation to falsehood and the possibility of fictitious claims against the deceased. Where, as in the instant case, the purpose of the oral testimony is to prove a lesser claim than what might be warranted by clear written evidence, to avoid prejudice to the estate of the deceased, the law has certainly no reason for its application. What is the latin maxim in that case? Ratione cessante, cessat ipsa lex.

With respect to the THIRD ELEMENT: Ong Chua vs. Carr? 1. An administrator who brings an action in behalf of the estate; 2. The action is brought by the heirs of a deceased plaintiff who were substituted for the latter. With respect to the FOURTH ELEMENT: 1. Negative testimony, that is testimony that a fact did not occurring during the lifetime of the deceased; 2. Testimony on the present possession by the witness of a written instrument signed by the deceased. To whom does the rule apply? To a witness of the plaintiff who attempts to testify on matters of fact occurring before the death of such deceased person or before such person became of unsound mind.

Facts: Henry Teck and his wife, Magdalena Lim sold a property to the plaintiff, Ong Chua. Chua executed a public document granting to the spouses the right to for the sum of P6, 500 within four years from the date of purchase. Later, Edward Carr went to the office of a practicing lawyer, Moore and sought the advice and assistance of the latter in regard to purchasing coconut lands. Moore then called Carr's attention to the lots above-mentioned and told him that he could buy the lots for P20, 000, the amount which Chua paid for them to Teck and Lim. Moore informed Carr that Teck and his wife had the right to repurchase the property in question from Chua and that such rights would expire in June, 1927. Both Chua and Carr requested Moore to draw the deed of sale of the property from Chua to Carr. Before the drafting of the deed, Chua stated to Moore that he consented to sell the properties to Carr on the condition that the sale should be subject to the rights of Teck and Lim to have the property reconveyed to them and that said rights were to be respected by the

Layno. Rabajante. Morana. Reyes. Suyat. Baraoidan|

27

2011-2012

[JARA POWER NOTES ON EVIDENCE]

vendee. According to Moore's own testimony, Carr was fully aware of those rights even before the execution of the deed, December 14, 1925, and that he consented to embody stipulations to the effect in said deed. Moore told Carr that the deed of sale could be made in such a form that Carr's title to the property purchased would appear to be absolute but that Carr was to bear in mind that the rights of Teck and Lim still existed and that the deed and other documents must be left in his, Moore's, possession until the expiration of the term for the right of repurchase and that, if the deed were made in that form, the loan of P6,500 (Carr‘s deficiency in the purchase price) could be obtained. The deed of sale was prepared without including therein the condition that sale was subject to Teck's and Lim's rights to repurchase. The deed was signed by Chua and duly acknowledged before Moore as notary public. Carr paid only P13, 500 in cash and promised, in writing, to pay to the vendor the balance of the purchase price. In July, 1926, Teck offered to repurchase the property in question from Chua who thereupon demanded of Carr the reconveyance of the property to the spouses, but Carr refused to do so, claiming that he had an absolute title to said property, and Chua then learned, for the first time, that the deed in question contained no reference to the rights of Teck and Lim to repurchase the property. On July 23, 1926, this action was brought with the plaintiff demanding that the deed in question be reformed in accordance therewith. Subsequent to the filling of the answer, Carr died, and the administrator of his estate was substituted as defendant. Upon such facts the court below ordered the reformation of the deed in accordance with the plaintiff's demand. Hence, this appeal. Issue: WON the court erred in permitting the plaintiff, Chua, to testify, over the defendant's objections, to fact occurring prior to the death of the defendant Carr Held: The rules bar parties to an action or proceeding against an executor or administrator or other representative of a deceased person upon a claim or demand against the estate of such deceased person from testifying as to any matter of fact occurring before the death of such deceased person. But it has generally been

given a liberal construction to promote justice, and it is held that it never was intended to serve as a shield for fraud. As stated in Jones on Evidence: The evidence of an adverse party is absolutely excluded by an independent, affirmative enactment making him incompetent as to transactions or communications with a deceased or incompetent person. These statutes, however, do not render the adverse party incompetent to testify to fraudulent transactions of the deceased, as the statutes are not designed to shield wrongdoers but the courts compel the adverse party to clearly establish the alleged fraudulent acts before admitting such testimony. In this case, a number of credible witnesses testified to facts which conclusively showed that Carr's conduct was tainted with fraud. The plaintiff did not take the witness stand until after the existence of fraud on the part of Carr and been established beyond a doubt and not by a mere preponderance of evidence. In these circumstances, we cannot hold that the trial court erred in not excluding the plaintiff's testimony. Important matters to discuss in the case: 1. Nature of the dispute 2. Whether or not the case fell under the parol evidence rule. 3. The reasons why there was a need to reform 4. Would it have been different if fraud was proven preliminarily? What is Filial Privilege? No person may be compelled to testify against his parents, other direct ascendants, children, or other direct descendants. In relation to the Family Code? Under the Family Code, the descendant may be compelled to testify against his parents and grandparents, if such testimony is indispensable in prosecuting a crime against the descendant or by one parent against another. Does this rule apply to both civil and criminal cases? No. only in civil cases. See modifications made under the family code.

Layno. Rabajante. Morana. Reyes. Suyat. Baraoidan|

28

2011-2012

[JARA POWER NOTES ON EVIDENCE]

Can modes of discovery be instead used if not allowed to testify? No. Not even depositions may be used as the same constitutes testimonial evidence. If the plaintiff makes use of the same depositions as evidence, may it be admitted in court? According to Hans Morana, no. Because the same was made in violation of the rules. What are privileged communications found under the Rules of Court? Sec. 24 deals with types of disqualifications by reason of privileged communication, to wit: 1. Communication between husband and wife; 2. Communication between attorney and client; 3. Communications between physician and patient; 4. Communications between a priest and a penitent; 5. Public officers and public interest. Is the enumeration exclusive? NO. there are other privileged matters not mentioned under Sec. 130 such as: 1. Editors may not be compelled to disclose the source of published news; 2. Voters may not be compelled to disclose for whom they voted; 3. Trade secrets; 4. Information contained in tax census returns; 5. Bank deposits; 6. Statements made in labor conciliation proceedings; 7. Communications of suspicious transactions to AMLC under the AMLA. What is the common element among such privileged communications? The core element in the said enumeration is the confidence reposed by the person giving such information to another who receives the same.

In marital privilege, what is the standard in determining whether the utterance was given in confidence? The expression ―any communication‖ means confidential communications during marital relationship. The essence of the privilege is to protect confidences only. And this must be true, because there can be no reason arising out from public policy, or otherwise, requiring that every word spoken between the husband and wife shall be privileged, irrespective of the presence in which spoken or the subject or occasion thereof. The spirit of the rule is that the privilege shall be construed to embrace only the knowledge which the husband or wife obtains from the other, which, but for the marriage relation and the confidence growing out of it, would not have been communicated, or which is of such nature or character as that, to repeat the same, would tend to unduly embarrass, or disturb the parties in their marital relations. What if the same communication overheard by a third person?

was

If a confidential communication made by one spouse to the other is overheard by a third person, the communication does not cease to be confidential between the spouses, and neither of them can testify without the consent of the other. But the legal prohibition to testify is directed to the spouses only, and accordingly, the third person overhearing cannot be prevented from testifying. For the same reason, where a privileged communication from one spouse to another comes into the hands of a third party, the latter may testify. But if the third party comes into possession of the communication by collusion and voluntary disclosure on the part of either of the spouses, he thereby becomes an agent of such spouse and cannot testify without the consent of the latter. What is the reason for the privilege? Society has a deeply-rooted interest in the preservation of peace of families and in the maintenance of the sacred institution of marriage, and its strongest safeguard is to preserve with zealous care any violations of those hollowed confidences inherent in, and inseparable from the marital status.

Layno. Rabajante. Morana. Reyes. Suyat. Baraoidan|

29

2011-2012

[JARA POWER NOTES ON EVIDENCE]

Can privileged communication be used to quash a subpoena?

refuse to divulge the name or identity of his client.

No. Sec. 4 of Rule 21 provides for exclusive grounds to quash a subpoena:

1) the court has a right to know that the client whose privileged information is sought to be protected is flesh and blood.

Quashing a Subpoena Duces Tecum: 1. The subpoena is unreasonable and oppressive; 2. The relevancy of the books, documents, or things does not appear; 3. If the person in whose behalf the subpoena is issued fails to advance the reasonable cost of the production thereof. Quashing a Subpoena ad Testificandum: 1. Witness is not bound thereby; 2. Witness fees and kilometrage allowed by these rules were not tendered when the subpoena was served. Furthermore, the marital disqualification rule does not provide for an absolute disqualification. The witness may still be summoned by the court but he is may be prohibited only to testify on matters that are covered by the marital privilege rule. For whose benefit is the attorney client privilege? It is a prohibition made against a counsel for the benefit of his lawyer. What happened in the case of Regala vs. Sandiganbayan? PCGG want to build up their case against Eduardo Coujuanco for the anomalies in the COCO LEVY FUNDS. PCGG wants petitioners divulge that Cojuangco indeed was a client of their firm, as well as other information regarding Cojuangco. Issue: Can the PCGG compel petitioners to divulge its client‘s name? Held: NO. As a matter of public policy, a client‘s identity should not be shrouded in mystery. The general is that a lawyer may not invoke the privilege and

2) the privilege begins to exist only after the attorney-client relationship has been established. The attorney-client privilege does not attach until there is a client. 3) the privilege generally pertains to the subject matter of the relationship. Finally, due process considerations require that the opposing party should, as a general rule, know his adversary. ―A party suing or sued is entitled to know who his opponent is.‖ He cannot be obliged to grope in the dark against unknown forces. Except: 1) Client identity is privileged where a strong probability exists that revealing the client‘s name would implicate that client in the very activity for which he sought the lawyer‘s advice. 2) Where disclosure would open the client to civil liability, his identity is privileged. 3) Where the government‘s lawyers have no case against an attorney‘s client unless, by revealing the client‘s name, the said name would furnish the only link that would form the chain of testimony necessary to convict an individual of a crime, the client‘s name is privileged. That client identity is privileged in those instances where a strong probability exists that the disclosure of the client's identity would implicate the client in the very criminal activity for which the lawyer‘s legal advice was obtained. The case was decided by a divided Supreme Court and therefore could not be considered a doctrinal pronouncement. What do you think was the big problem? The main contention of the dissenters was that the rule on attorney-client privilege cannot be used to hide a crime and the anomalies involving the coco-levy funds involved a crime. However, the point was resolved by the majority in this light: the client was still in the process of discussing business matters with the law firm.

Layno. Rabajante. Morana. Reyes. Suyat. Baraoidan|

30

2011-2012

[JARA POWER NOTES ON EVIDENCE]

No crime has yet to be committed since the same was in an indeterminate stage. Unless it becomes clear that the matters confided upon the lawyers of the client constitute a crime, ACCRA had no obligation to divulge information regarding the same. ADMISSIONS AND CONFESSIONS What is a self-serving statement? It refers to one which has been made extrajudicially by the party to favor his interest. What is a disserving statement? A disserving statement or an admission is any extra-judicial statement or conduct (act or omission) by a party that is inconsistent with the position the party presently takes. Declaration that “I am a tenant of this house.” Is that an admission? It depends. See 2004 Jara Notes for a clearer discussion. What are the requisites for an admission to be admissible? 1. It must involve matters of fact and not of law; 2. Be categorical and definite; 3. Be knowingly and voluntarily made; 4. Be adverse to the admitter‘s interest; otherwise it would be self-serving and inadmissible. Why is an admission admissible while a selfserving statement not admissible? Man cannot make evidence for himself. The reason for the rule is that what a man says against his own interest may be safely believed; but it is not safe to credit him where he is advocating his interest. If a statement favorable to the interests of the witness is uttered in court, is it still selfserving? Not anymore. Self-serving statements are allowed to be made in court. Malamang kelangan mo ipaglaban yung kaso mo.

Give an example of a self-serving statement WARNING: make sure na kumpleto lahat ng elements niyo kapag nagbigay kayo ng example. SHOTGUN QUESTION ETO. What happened in the case of Estrada vs. Desierto? Doctrine: The Angara diary is admissible in evidence. It partakes the nature of an adoptive admission. An adoptive admission is a party‘s reaction to a statement or action by another person when it is reasonable to treat the party‘s reaction as an admission of something stated or implied by the other person. What happened in People vs. Holgado? Facts: Sisenando Holgado and Filomeno Morales had disputes about the occupation of certain land situated in the municipality of Pinamalayan, Province of Mindoro. On the morning of June 15, 1927, the two men happened to meet. The argument was renewed, and they agreed to fight. They did engage in a bolo duel with a fatal result for Filomeno Morales, who was killed almost instantly. Sisenando Holgado was also seriously wounded but was able to proceed to a neighboring house. From there Sisenando Holgado was taken to the municipal building where he made a sworn statement before the municipal president, in which he declared that only he and Filomeno Morales fought. About one month later, Sisenando Holgado died from the wounds received in the fight. The disputable point is whether the accused Eugenio Toledo intervened in the quarrel and dealt a mortal blow to Filomeno Morales. For the prosecution, there was presented the witness Justina Villanueva, the querida of Filomeno Morales, who testified to the presence and participation of Eugenio Toledo. Her testimony was partially corroborated by that of the witness Justina Llave. On the other hand, the theory for the defense was that Toledo was in another place when the fight between Morales and Holgado occurred and that his only participation was on meeting Holgado, who was his landlord or master, in helping him to a nearby house. To this effect is the testimony of the accused and of Conrado Holgado, the son of Sisenando Holgado. The defense also relied upon the affidavit of Sisenando Holgado, Exhibit 1, which

Layno. Rabajante. Morana. Reyes. Suyat. Baraoidan|

31

2011-2012

[JARA POWER NOTES ON EVIDENCE]

was identified by the municipal president of Pinamalayan. Issue: WON the declaration of a third person against his own penal interest is covered by the exceptions to the hearsay rule. Held: Yes. In the case of Pace vs. State ([1911], Court of Criminal Appeals of Texas, 135 Southwestern, 379), the appellant offered to prove in the trial court by the witness Byron Kyle that on Saturday morning following the killing of the deceased on the previous Sunday he had a conversation with Dick Cain, one of the parties to the homicide, in which Dick Cain admitted the he killed the deceased. The court ruled: . . . Wherever the state seeks to fasten criminality upon the party on trial, the accused had a right to meet and rebut any testimony which may be offered against him in any legitimate way. If Cain had been upon trial, his confession to the witness Kyle would have been admissible beyond any shadow of doubt, and would have been upon trial, his confession to the witness Kyle would have been admissible beyond any shadow of doubt, and would have been strong evidence to go before the jury. The estate would have been seeking to introduce this and with great earnestness, and correctly so. If appellant could prove that another party or others committed the homicide, it might prove his innocence, and would be strong evidence to go before the jury in his favor. Any legitimate fact or circumstance which would meet or tend to meet the state's case and break the force of criminative facts introduced against the accused is always admissible. Appellant's contention was that he did not kill the deceased, but that Cain did. The state's theory was the appellant shot the deceased, and Cain did not shoot him. Under the rules of evidence this testimony was clearly inadmissible. We would like finally to turn attention to what was said by the editor of L. R. A. in his note in volume 37 hereinbefore referred to, viz: The purpose of all evidence is to get at the truth. The reason for the hearsay rule is that the extrajudicial and unsworn statement of another is not the best method of serving this purpose. In other words, the great possibility of the fabrication of falsehoods, and the inability to prove their untruth, requires that the doors be closed to such evidence. So long therefore as a

declarant is available as a witness, his extrajudicial statement should not be heard. Where, however, the declarant is dead or has disappeared, his previous statements, out of court, if not inadmissible on other grounds, are the best evidence. But they are not rendered inadmissible by the mere fact that the declarant is unavailable, - something else is necessary. One fact which will satisfy this necessity is that the declaration is or was against the declarant's interest, and this is because no sane person will be presumed to tell a falsehood to his own detriment. xxx

xxx

xxx

Again, if, seems indisputable, the desire to close the door to falsehood which cannot be detected dictates the exclusion of such testimony, the question as to the effect to be given to such a confession is solely one of weight and credibility. ... Any man outside of a court and unhampered by the pressure of technical procedure, unreasoned rules of evidence, and cumulative authority, would say that if a man deliberately acknowledged himself to be the perpetrator of a crime and exonerated the person charged with the crime, and there was other evidence indicative of the truthfulness of the statement, the accused man should not be permitted to go to prison or to the electric chair to expiate a crime he never committed. Shall Judges trained and experienced in the law display less discerning common sense that the layman and allow precedent to overcome truth? Adoptive Silence

admission

Adoptive Admission A party‗s reaction to a statement or action by another person when it is reasonable to treat the party‗s reaction as an admission of something stated or implied by the other person

vs.

Admission

by

Admission by Silence An act or declaration made in the presence and within the hearing or observation of a party who does or says nothing when the act or declaration is such as naturally to call for action or comment if not true, and when proper and possible for him to do so.

Layno. Rabajante. Morana. Reyes. Suyat. Baraoidan|

32

2011-2012

[JARA POWER NOTES ON EVIDENCE]

What is the principle of Res Inter Alios Acta? The res inter alios acta rule ordains that the rights of a party cannot be prejudiced by an act, declaration, or omission of another. An extrajudicial confession is binding onlyupon the confessant and is not admissible against his coaccused. the reason for the rule is that, on a principle of good faith and mutual convenience, a man‘s own acts are binding upon himself, and are evidence against him. So are his conduct and declarations. Yet it would not be rightly inconvenient, but also manifestly unjust, that a man should be bound by the acts of mere unauthorized strangers; and if a party ought not to be bound by the acts of strangers, neither ought their acts or conduct be used as evidence against him (People v. Raquel, G.R. No. 119005).

What are the requisites for one to be a state witness? 1. There is absolute necessity for the testimony of the accused whose discharge is requested; 2. There is no other direct evidence available for the proper prosecution of the offense committed, except the testimony of the said accused; 3. The testimony of said accused can be substantially corroborated in its material points; 4. Said accused does not appear to be the most guilty; and 5. Said accused has not at any time been convicted of any offense involving moral turpitude. Should a motion be filed?

What are the two branches of the Res Inter Alios Acta Rule? 1. The rule that the rights of a party cannot be prejudiced by an act, declaration, or omission of another; 2. The rule that evidence of previous conduct or similar acts at one time is not admissible to prove that one did or did not do the same act at another time.

Yes. A motion must be filed by the prosecution before it rests its case. Does it have to be heard ex-parte? NO. before resolving such motion to discharge the accused to be a state-witness, the court shall require the prosecution to present evidence and the sworn statement of the proposed state witness at a hearing of the motion to discharge.

What are the exceptions to the rule? 1. Admission by a co-partner or agent; 2. Admission by a co-conspirator; 3. Admission by privies. If a person takes the witness stand and makes an admission prejudicial to a third person, is the rule on “res inter alios acta” still applicable? NO. Statements made in open court by a witness implicating a person aside from his own judicial admissions, are admissible as declarations from one who has personal knowledge of the facts testified thereto.

As prosecutor, how do you plan to convince the court to allow the accused to testify? You convince the court by complying with all the requisites mandated by the law for one to become state witness. More importantly, you have to prove that the accused applying to become a state-witness must not be the most guilty of the crime charged. What assurance can you give the proposed state-witness in case he testifies? If the court denies the motion for discharge of the accused as state witness, his sworn statement shall be inadmissible as evidence.

Who is a state witness? He is one of two or more persons jointly charged with the commission of a crime but who is discharged with his consent as such accused so that he may be a witness for the State

Discuss the procedure for one to become a witness for purposes of RA 6981. MEMAID WORK LANG ITO.

Layno. Rabajante. Morana. Reyes. Suyat. Baraoidan|

33

2011-2012

[JARA POWER NOTES ON EVIDENCE]

What is the assurance that the accused will testify for the state? Any witness admitted into the program of the Witness Protection, Security and Benefit Act cannot refuse to testify or give evidence or produce books, documents, records or writings necessary for the prosecution of the offense or offenses for which he has been admitted into the Program on the ground of the constitutional right against self-incrimination but he shall enjoy immunity from criminal prosecution and cannot be subjected to any penalty or forfeiture for any transaction, matter or thing concerning his compelled testimony or books, documents, records and writings produced (Sec. 14, R.A. 6981). Who may be admitted to the Witness Protection, Security and Benefit Program? Any person who has witnessed or has knowledge or information on the commission of a crime and has testified or is testifying or about to testify before any judicial or quasi-judicial body, or before any investigating authority may be admitted provided that: 1. the offense in which his testimony will be used is a grave felony as defined under the Revised Penal Code, or its equivalent under special laws; 2. his testimony can be substantially corroborated in its material points; 3. he or any member of his family within the second civil degree of consanguinity or affinity is subjected to threats to life or bodily injury or there is a likelihood that he will be killed, forced, intimidated, harassed or corrupted to prevent him from testifying, or to testify falsely, or evasively, because or on account of his testimony; and 4. he is not a law enforcement officer, even if he would be testifying against the other law enforcement officers. In such a case, only the immediate members of his family may avail themselves of the protection provided for under the Act (Sec. 3, R.A. 6981).

If the person does not comply with the conditions, may the sworn statement be used against him? (Sworn statement of a person who refuses to testify thereon) Yes, if he fails or refuses to testify or to continue to testify without just cause when lawfully obliged to do so, he shall be prosecuted for contempt. If he testifies falsely or evasively, he shall be liable to prosecution for perjury. If a State witness fails or refuses to testify, or testifies falsely or evasively, or violates any condition accompanying such immunity without just cause, as determined in a hearing by the proper court, his immunity shall be removed and he shall be subject to contempt or criminal prosecution. Moreover, the enjoyment of all rights and benefits under R.A. 6981 shall be deemed terminated. The witness may, however, purge himself of the contumacious acts by testifying at any appropriate stage of the proceedings (Sec. 13, R.A. 6981). If in a criminal case the accused pleads guilty, can he be convicted of the crime charged? YES. No need to present evidence on the part of the prosecution. How may an accused change his plea? At any time before the judgment of conviction becomes final, the court may permit an improvident plea of guilty to be withdrawn and be substituted by a plea of not guilty. Can the prosecutor make use of the withdrawn plea as an evidence against him? No. a plea of guilty later withdrawn, or an unaccepted offer of a plea of guilty to a lesser offense, is not admissible in evidence against the accused who made the plea or offer. Is the same applicable in a civil case? If so, how can you do it? Yes. Such can be done by amending a party‘s pleadings. Upon admissions made in an answer – he can amend the admissions in the answer into a specific denial.

Layno. Rabajante. Morana. Reyes. Suyat. Baraoidan|

34

2011-2012

[JARA POWER NOTES ON EVIDENCE]

RULE NO DNA EVIDENCE What is DNA? DNA (deoxyribonucleic acid) is the chain of molecules found in every nucleated cell of the body (Sec. 3, Rule on DNA Evidence). It is the fundamental building block of a person‘s entire genetic make-up, which is found in all human cells and is the same in every cell of the same person (People v. Umanito, G.R. No. 172607, Oct. 26, 2007). What is DNA evidence? It constitutes the totality of the DNA profiles, results and other genetic information directly generated from DNA testing of biological samples (Sec. 3). What is DNA testing? It means verified and credible scientific methods which include the extraction of DNA from biological samples, the generation of DNA profiles and the comparison of the information obtained from the DNA testing of biological samples for the purpose of determining, with reasonable certainty, whether or not the DNA obtained from two or more distinct biological samples originates from the same person (direct identification) or if the biological samples originate from related persons (Kinship Analysis). May DNA testing be conducted absent a prior court order? Yes. The Rules on DNA Evidence does not preclude a DNA testing, without need of a prior court order, at the behest of any party, including law enforcement agencies, before a suit or proceeding is commenced (Sec. 4). What are the requisites for the issuance of a DNA testing order? In pending actions, the appropriate court may, at any time issue a DNA testing order either motu proprio or upon application of any person who has a legal interest in the matter in litigation after due hearing and notice to the parties and upon showing of the following: 1. A biological sample exists relevant to the case;

that

is

2. The biological sample: 3. was not previously subjected to the type of DNA testing now requested; or 4. was previously subjected to DNA testing, but the results may require confirmation for good reasons; 5. The DNA testing uses a scientifically valid technique; 6. The DNA testing has the scientific potential to produce new information that is relevant to the proper resolution of the case; and 7. The existence of other factors, if any, which the court may consider as potentially affecting the accuracy or integrity of the DNA testing (Sec. 4). Is the order granting the DNA testing appealable? No. An order granting the DNA testing shall be immediately executory and shall not be appealable. Any petition for certiorari initiated therefrom shall not, in any way, stay the implementation thereof, unless a higher court issues an injunctive order (Sec. 5). During Alexis’ trial for rape with murder, the prosecution sought to introduce DNA evidence against him, based on forensic laboratory matching of the materials found at the crime scene and Alexis’ hair and blood samples. Alexis’ counsel objected, claiming that DNA evidence is inadmissible because the materials taken from Alexis were in violation of his constitutional right against self-incrimination as well as his right of privacy and personal integrity. Should the DNA evidence be admitted or not? Reason. The DNA evidence should be admitted. It is not in violation of the constitutional right against selfincrimination or his right of privacy and personal integrity. The right against self-incrimination is applicable only to testimonial evidence. Extracting a blood sample and cutting a strand from the hair of the accused are purely mechanical acts that do not involve his discretion nor require his intelligence. Is the result of DNA testing automatically admitted as evidence in the case in which it was sought for? No. The grant of a DNA testing application shall not be construed as an automatic admission into

Layno. Rabajante. Morana. Reyes. Suyat. Baraoidan|

35

2011-2012

[JARA POWER NOTES ON EVIDENCE]

evidence of any component of the DNA evidence that may be obtained as a result thereof (Sec. 5).

What is the remedy of the convict if the postconviction DNA testing result is favorable to him?

If a DNA test was conducted, what are the possible results that it may yield?

The convict or the prosecution may file a petition for a writ of habeas corpus in the court of origin. In case the court, after due hearing, finds the petition to be meritorious, it shall reverse or modify the judgment of conviction and order the release of the convict, unless continued detention is justified for a lawful cause.

1. The samples are similar, and could have originated from the same source (Rule of Inclusion). In such a case, the analyst proceeds to determine the statistical significance of the similarity. 2. The samples are different hence it must have originated from different sources (Rule of Exclusion). This conclusion is absolute and requires no further analysis; 3. The test is inconclusive. This might occur due to degradation, contamination, failure of some aspect of protocol, or some other reasons. Analysis might be repeated to obtain a more conclusive result (People v. Vallejo, G.R. No. 144656, May 9, 2002). What should the courts consider evaluating DNA testing results?

in

1. The evaluation of the weight of matching DNA evidence or the relevance of mismatching DNA evidence; 2. The results of the DNA testing in the light of the totality of the other evidence presented in the case; and 3. DNA results that exclude the putative parent from paternity shall be conclusive proof of non-paternity (Sec. 9) To whom is post-conviction DNA testing available? Post-conviction DNA testing may be available, without need of prior court order, to the prosecution or any person convicted by final and executory judgment. What are the requisites for the applicability of the Post-conviction DNA testing? 1. Existing biological sample; 2. Such sample is relevant to the case; and 3. The testing would probably result in the reversal or modification of the judgment of conviction (Sec. 6).

What should the courts consider in determining the probative value of DNA evidence? 1. The chain of custody, including how the biological samples were collected, how they were handled, and the possibility of contamination of the samples; 2. The DNA testing methodology, including the procedure followed in analyzing the samples, the advantages and disadvantages of the procedure, and compliance with the scientifically valid standards in conducting the tests; 3. The forensic DNA laboratory, including accreditation by any reputable standards-setting institution and the qualification of the analyst who conducted the tests. If the laboratory is not accredited, the relevant experience of the laboratory in forensic casework and credibility shall be properly established; and 4. The reliability of the testing result (Sec. 7). What are the things to be considered in assessing the probative value of DNA evidence? 1. How the samples are collected; 2. How they were handled; 3. The possibility of the contamination of the samples; 4. The procedure followed in analyzing the samples; 5. Whether the proper standards and procedures were followed in conducting the tests; and 6. The qualification of the analyst who conducted the tests. (Ibid.)

Layno. Rabajante. Morana. Reyes. Suyat. Baraoidan|

36

2011-2012

[JARA POWER NOTES ON EVIDENCE]

What are the things to be considered in evaluating whether or not the DNA testing methodology is reliable? 1. The falsifiability of the principles or methods used, that is, whether the theory or technique can be and has been tested; 2. the subjection to peer review and publication of the principles or methods; 3. The general acceptance of the principles or methods by the relevant scientific community; 4. The existence and maintenance of standards and controls to ensure the correctness of data generated; 5. The existence of an appropriate reference population database; and 6. The general degree of confidence attributed to mathematical calculations used in comparing DNA profiles and the significance and limitation of statistical calculations used in comparing DNA profiles.

HEARSAY EVIDENCE What is Sec. 36? A witness can testify only to those facts which he knows of his personal knowledge; that is, which are derived from his own perception, except as otherwise provided in these rules. Are there any exceptions to the hearsay rule? 1. 2. 3. 4. 5. 6. 7. 8. 9. 10. 11.

What Kind of Evidence is DNA Evidence? 12. It can be object, documentary, or testimonial, depending on what you present in court. It is not exclusively classifiable. In post-conviction DNA testing, what does “conviction” mean? Conviction means a person has been convicted by final judgment. Even if he is already sentenced by final judgment, he may still avail of such remedy. Does the rule on post-conviction DNA testing violate the rule on immutability of judgment? No. Once post-conviction testing turns out to be favorable to the accused, it has the same effect as annulling the court‘s judgment. If that is the case, then why is the remedy a petition for Habeas Corpus? Why not an action for annulment of judgment under Rule 47? An action for habeas corpus has the same effect as that of an action for annulment of judgment. The difference is that according to Jara, Rule 47 is only available to civil cases. Habeas Corpus may be availed of in criminal cases.

13.

dying declaration declaration against interest act or declaration about pedigree family reputation or tradition regarding pedigree common reputation res gestae entries in the ordinary course of business entries in official records commercial lists learned treatises testimony or deposition at a former proceeding Sec. 28 of the Rules on Examination of a Child Witness Rule 8 of the Rules on Electronic Evidence

Can a documentary evidence be hearsay? YES. The one who knows the contents of the document must testify in court and must have personal knowledge of the facts stated therein. When a document is presented in court, there is a presumption that someone has prepared the said document. In order to properly submit the same, the person who prepared it or who had a part in the execution of the said document must testify as to the contents of the same. If it the said document was merely presented to the court without giving the adverse party any opportunity to cross examine the person who prepared or who was part of the preparation of the said document, then the same document constitutes hearsay evidence. Is hearsay evidence relevant? Why should we exclude such relevant evidence? Hearsay evidence is relevant because it may have a reasonable tendency to prove the facts in issue. However, it is not competent; it is

Layno. Rabajante. Morana. Reyes. Suyat. Baraoidan|

37

2011-2012

[JARA POWER NOTES ON EVIDENCE]

expressly excluded by the Rules. The reason for its exclusion is the absence of the opportunity to cross-examine the witness testifying before the court. The veracity of the facts cannot be tested. Opportunity to cross-examine What are the two evidence?

aspects of

hearsay

1. Testimonial evidence that was derived from personal knowledge; 2. Testimony of a witness that may derived from personal knowledge did not give the adverse party opportunity to cross-examine witness.

Supposing it is the other way around? If it is the witness who testifies according to his own personal knowledge? Is it possible that it can be hearsay? (Jara 2004 Notes) Yes. There is still hearsay evidence if the witness testifies according to his own personal knowledge, BUT his testimony was not subjected to cross-examination.

not be but an the

Can you not cross-examine the witness who offered the hearsay evidence? YES. OPINION (HPM): In fact, you can ask anything during cross examination (generally, we follow the ENGLISH RULE), therefore, there is nothing wrong if you cross-examine the witness. You can cross-examine him on how he acquired the hearsay knowledge, or those independently relevant statements. See also the purposes of cross-examination, and it does not only include testing the truthfulness of the statement but also to discredit the witness. Hence, even if the testimony is hearsay, you can attack the credibility of the witness during crossexamination. However, it is submitted that once you cross-examine a witness who testified on hearsay, the adverse party is deemed to have waived his right to object on its admissibility. The objection must be raised during the direct exam, hence, if the counsel was able to object, then there is no necessity for him to cross-examine. OPINION (IPL): You could cross-examine the witness who offered the hearsay testimony. WHAT YOU CANNOT CROSS-EXAMINE IS THE PERSON WHO MADE THE OUT-OFCOURT STATEMENT. That is the problem the hearsay rule wants to address. It is because of the above reason that if the affiants of affidavits do not take the witness stand to affirm their averments in their affidavits, such affidavits must be excluded from the judicial proceeding, being inadmissible hearsay.

NOTE: If failure of the witness to return to court is due to the act of the adverse party, then such testimony does not become hearsay. What is the Rule on Independently Relevant Statements? Is it hearsay? The doctrine provides that a witness ma testify to the statements made by a person if, for instance, the fact that such statements were made by the latter would indicate the latter‘s mental state or physical condition. Such statements are relevant since the statements made are the very facts in issue or circumstantial evidence of the facts in issue. It is not hearsay evidence. It may have certain characteristics of hearsay, but its application is not hearsay. It is DIRECT EVIDENCE. What are the different classifications of Independently Relevant Statements? 1. Those statements which are the very facts in issue; and 2. Those statements which are circumstantial evidence of the facts in issue. Does the hearsay rule apply to summary procedure? Is there an opportunity to crossexamine in summary procedure? YES. Although in both civil and criminal cases the direct testimonies of the witnesses shall be contained in their affidavits, the accused in criminal cases covered by Summary Procedure has the right to cross-examine the witnesses. However, no such right is available to the parties in civil cases. But see Sec. 20 of the Rule on Summary Procedure (which is applicable to both civil and criminal) such that, hearsay statement shall subject witness to disciplinary proceedings and expunge the statements off the record.

Layno. Rabajante. Morana. Reyes. Suyat. Baraoidan|

38

2011-2012

[JARA POWER NOTES ON EVIDENCE]

People vs. Cloud At around 11:00 o‘clock in the morning on August 2, 1988 while a certain Mrs. Josephine Aguilar was at the emergency room of St. Luke‘s Hospital, Quezon City to have some stitches removed from her daughter‘s head her attention was called by a limpid boy being carried by a man followed by an old woman who was shouting hysterically. The boy is John Albert Cloud. She noticed that the face of the boy was swollen and bruised and his body covered with dry blood. A nurse commented that the little boy – not more than three years old – must have been hit by a truck (tsn, J. Aguilar, June 21, 1993, pp. 7-10, 14-15, 33). But the words of the old woman – the lola - of the little boy, showed the cause of the injury to be otherwise for she was repeatedly saying in a potpourri of cries and tears: ―Pinatay siya nf sariling ama!‖ The old woman told the people inside the Emergency Room that the boy‘s father – Robert Cloud – wouldn‘t allow John Albert to come with her and when the boy started to cry and wouldn‘t stop crying his father began to beat the boy hard, tied his hands, and made ―tusok, tusok‖ in his body. The father continued beating the boy even when excrements were already coming out from the boy‘s anus (tsn. J Aguilar, June 21, 1993, pp. 12-13, 22). The male companion of the boy said to the old woman: ―Hoy, tigil ka na!‖ ―Wag kang maingay.‖ And told the people at E.R.: ―Sira and ulo ng matanda, eh!‖ (tsn, J Aguilar, July 12, 1993, pp. 8-9). But the old woman wouldn‘t stop and continued to say: ―Putang-ina ang ama niya . . . Hayop siya!‖

Cloud and family left his house at No. 69 San Isidro Street, barangay Sto. Niño, Quezon City[;] the boy‘s body was brought to Rey Funeral Homes[;] Dr. E. Cacas certified that the cause of death of John Albert Cloud is broncho pneumonia with heart complications (exh. D-48) [;] and that the autopsy on the cadaver was waived by Natividad Calpito Cloud who claimed to be the boy‘s mother per her ―Affidavit‖ dated August 3, 1988 (Exh. ―D-47). Atty Balbin thereafter contacted the NBI and requested for the exhumation of the boy‘s cadaver. Issue: WON the statements of Josephine Aguilar are admissible as evidence. Held: Yes. The trial court was of the opinion that what Ms. Aguilar heard or saw does not merely constitute an independently relevant statement which it considered as an ―exception to the hearsay rule, only as to the tenor rather than the intrinsic truth or falsity of its contents.‖[18] We will clarify this. Insofar as the statements of Rufina Alconyes are concerned, they are admissible as part of the res gestae they having been caused by and did result from the startling, if not gruesome, occurrence that she witnessed; and these were shortly thereafter uttered by her with spontaneity, without prior opportunity to contrive the same. The report made thereof by Josephine Aguilar is not hearsay since she was actually there and personally heard the statements of Alconyes which she recounted in court. Her account of said statements of Alconyes are admissible under the doctrine of Aindependently relevant statements, with respect to the tenor and not the truth thereof, since independent of the truth or falsity of the same they are relevant to the issue on the cause of the death of the victim.

When the doctor pronounced the boy dead the old woman knelt before him and cried like (Ix)ion (tsn, J. Aguilar, June 21, 1993, p. 10). His baptismal certificate says that John Albert was born on October 2, 1987 to Janet Villagracia and John Robert Cloud (Exh. ‗3‘).

Who was the Ponente?

The ear-piercing would probably have ended there but for the fact that Mrs. Aguilar‘s conscience was bothered by what she saw and heard as narrated above and decided to do something about it. She approached Atty. Remedios Balbin, Chairman in Quezon City of a civil liberties organization. Atty. Balbin, after a few weeks of research found out that Robert

The witness in this case testified as to the fact that a certain old lady was screaming invectives against a person who allegedly killed his son.

Justice Regalado The decision made use of independently relevant statements. How?

Layno. Rabajante. Morana. Reyes. Suyat. Baraoidan|

39

2011-2012

[JARA POWER NOTES ON EVIDENCE]

Does that mean that the prosecution can convict a person of a serious crime using only circumstantial evidence?

Why is it exempted?

YES, provided that:

What is the legal maxim that is usually quoted?

1. there is more than one circumstance; 2. the facts from which the inferences are derived are proven; and 3. the combination of all the circumstances is such as to produce a conviction beyond reasonable doubt. What do you mean by the term ante litem motam? In cases applicable, the statement must be prior to the controversy, but according to CJ Moran, it means that it must not only be prior to the suit but prior to any controversy even if a suit has not yet been instituted. What is the concept of a dying declaration? The declaration of a dying person, made under the consciousness of an impending death, may be received in any case wherein his death is the subject of inquiry, as evidence of the cause and surrounding circumstances of such death. Can we apply this to a criminal case? How about a civil case (HOW)? Commonly applicable to criminal cases since the subject of the statement is the death of the declarant. However, it is applicable to civil cases when the subject matter of the cases involves the death of the declarant, or that the death is a vital issue in the civil case. As long as the relevance is clear, a dying declaration may now be introduced in a criminal or a civil action and the relevance is satisfied when the subject of the inquiry is the death of the declarant himself.

Necessity and trustworthiness.

Truth sits on the lips of the dying man. What if the defense presents evidence that the declarant during his lifetime is a congenital liar? Is that a good strategy on the part of the defense? NO. That is not a good defense. It will not even have any effect at all. The rule on dying declarations does not require that the declarant be one of proven honesty. For as long as the essential elements of a dying declaration is present, the same may be admitted in evidence. Essential Elements/Requisites of a dying declaration? Requisites: 1. That death is imminent and the declarant is conscious of that fact; 2. That the declaration refers to the cause and the surrounding circumstances of such death; 3. That the declaration relates to the facts which the victim is competent to testify to; 4. That the declaration is offered in a case wherein the declarant‗s death is subject of the inquiry (the victim necessarily must have died); 5. That the statement is complete in itself (People v. De Joya, G.R. No. 75028, November 8, 1991); and 6. The declarant should have died. How do you prove to the court that the statements were made under the consciousness of an impending death?

Must a dying declaration be in writing? NO. The Revised Rules on Evidence do not require that a dying declaration must be made in writing to be admissible. Indeed, to impose such a requirement would be to exclude many statements from a victim in extremis for want of paper and pen at the critical moment (People v. Viovicente, G.R. No. 118707).

A declaration will be deemed as having been made under the consciousness of imminent death, in consideration of: 1. The words or statements of the declarant on the same occasion; 2. His conduct at the time the declaration is made;

Layno. Rabajante. Morana. Reyes. Suyat. Baraoidan|

40

2011-2012

[JARA POWER NOTES ON EVIDENCE]

3. The serious nature of his wounds as would necessarily engender a belief on his part that he would not survive therefrom. Can a dying declaration be used if the crime charged is Rape with Homicide? Only as to the death, i.e., the facts surrounding the death, but not the facts surrounding the rape. Jara: if the declarant says that he/she was raped, and subsequently dies, that cannot be considered a dying declaration.

Distinction of declaration against interest vs. admission? Admissions against interest are those made by a party to a litigation or by one in privity with or identified in legal interest with such party, and are admissible whether or not the declarant is available as a witness. Declarations against interest are those made by a person who is neither a part nor in privity with a party to the suit, are secondary evidence, but constitute an exception to the hearsay rule, and are admissible only when the declarant is unavailable as a witness. Give an example of a declaration against interest.

What if the victim does not die? Always remember DEATH IS AN ESSENTIAL ELEMENT. Why? If the declarant is alive, he can testify in court personally. It will not be considered as a dying declaration, but nevertheless, it may be admissible as part of a res gestae What if victim dies only after three (3) months? As long as he believed that he is going to die. What is s declaration against interest? The declaration made by a person deceased, or unable to testify, against the interest of the declarant, if the fact asserted in the declaration was at the time it was made so far contrary to declarant‘s own interest, that a reasonable man in his position would not have made the declaration unless he believed it to be true. Requisites? 1. That the declarant is dead or unable to testify; 2. That it relates to a fact against the interests of the declarant; 3. That at the time he made said declaration the declarant was aware that it was contrary to his aforesaid interest; and 4. That the declarant had no motive to falsify and he believed such declaration to be true.

Make sure to give an example both for a criminal case and a civil case, and remember na dapat kumpleto lahat ng elements. What are the evidenciary Rules on Pedigree? SEC. 39. Act or declaration about pedigree.— The act or declaration of a person deceased, or unable to testify, in respect to the pedigree of another person related to him by birth or marriage, may be received in evidence where it occurred before the controversy, and the relationship between the two persons is shown by evidence other than such act or declaration. The word "pedigree" includes relationship, family genealogy, birth, marriage, death, the dates when and the places where these facts occurred, and the names of the relatives. It embraces also facts of family history intimately connected with pedigree. SEC. 40. Family reputation or tradition regarding pedigree.— The reputation or tradition existing in a family previous to the controversy, in respect to the pedigree of any one of its members, may be received in evidence if the witness testifying thereon be also a member of the family, either by consanguinity or affinity. Entries in family bibles or other family books or charts, engraving on rings, family portraits and the like, may be received as evidence of pedigree.

Layno. Rabajante. Morana. Reyes. Suyat. Baraoidan|

41

2011-2012

[JARA POWER NOTES ON EVIDENCE]

Distinctions between Sec. 39 and Sec. 40.

What is the rule on Res Gestae?

Sec. 39 Act or declaration about pedigree.

Statements made by a person while a startling occurrence is taking place or immediately prior or subsequent thereto with respect to the circumstances thereof, may be given in evidence as part of the res gestae. So, also, statements accompanying an equivocal act material to the issue, and giving it a legal significance, may be received as part of the res gestae.

Declarant is deceased or unable to testify. Witness need not be a member of the family. Relation of the declarant and the person subject of the inquiry must be established by independent evidence

Sec. 40 Family reputation or tradition regarding pedigree. Declarant is the witness himself. Witness is a member of the family. The witness is himself the one to whom the fact relates, it is not necessary for him to establish by independent evidence his relationship to the family.

Give an example. Just give an example regarding your family, para mas medaling idefend. He grills a lot of students in examples. When were you born? Is that hearsay? So if it is hearsay, when you state that in court, the court will not believe you? the fact that you know your date of birth does not come from your own personal knowledge because at the time you were born you were completely ignorant or innocent. In fact, your information of your birth is derived from another source. Nevertheless, the information regarding your birth is admissible for the convenience of everybody. It is a matter of necessity. The birth certificate is also hearsay evidence, but because it is a public document. Documents consisting of entries in public records made in the performance of a duty by a public officer are prima facie evidence of the facts therein stated. All other public documents are evidence, even against a third person, of the fact which gave rise to their execution and of the date of the latter. Basis for admitting such evidence Necessity and Trustworthiness

Why are they hearsay? Because the one testifying in court is not the one who made the declaration. Why do we admit them? Necessity and Trustworthiness. There are other declarations which are admitted as original evidence, being distinguished from hearsay by their connection with the principal fact under investigation. The affairs of men consist of a complication of circumstances so intimately inter-woven as to be hardly separable from each other. They are trustworthy because the statements are made instinctively, while the declarant‘s mental power for deliberation in concocting matters are controlled and stilled by the shocking influence of a startling occurrence, so that all his utterances at the time are the reflex product of immediate sensual impressions, unaided by retrospective mental action. Necessity, because such natural and spontaneous utterances are more convincing than the testimony of the same person on the stand. Different classifications Declarations Verbal Acts The res gestae is the equivocal act. Verbal act must be contemporaneous with or must accompany the equivocal act.

of

Res

Gestae

Spontaneous Statements The res gestae is the startling occurrence. Statements may be made prior, or immediately after the startling occurrence.

Layno. Rabajante. Morana. Reyes. Suyat. Baraoidan|

42

2011-2012

[JARA POWER NOTES ON EVIDENCE]

What are Spontaneous statements?

doing it may be proved if they are becessary to understand it.

Statements made by a person while a startling occurrence is taking place, or immediately prior or subsequent thereto, with respect to the circumstances thereof. What is the occurrence?

concept

of

a

startling

As it is essential that the statements be natural and spontaneous, unreflected and instinctive, it is also essential that they should have been caused by something startling enough to produce nervous excitement, and to keept the will dormant so far as any deliberation in concocting matters for speech or selecting words is concerned. Example? A murder, a suicide, a railroad accident and the like. What if the statements were made for some period after the startling occurrence took place? Experience shows that a startling occurrence may extend its exciting influence over a subsequent period of time which may be long or short according to the relative gravity or seriousness of the startling occurrence. If the statements were made while the nervous excitement was still working on the declarant‘s mind, they are admissible; otherwise, they are not. How does one determine the spontaneity of the occurrence? 1. Lapse of time between the act and the declaration relating to it; 2. Whether there was an opportunity for fabrication or a likelihood of it; 3. The mental and physical condition of the declarant and the character of the occurrence; 4. The nature and form of the declaration. What are verbal acts? Statements accompanying an equivocal act material to the issue and giving it a legal significance. Statements accompanying and explaining that act made by or to the person

What are the requisites for verbal acts to be admissible? 1. There must be an equivocal act; 2. The equivocal act must be material to the issue; 3. The statement in question muset be necessary to understand the equivocal act; and 4. The statement must accompany the equivocal act. What is an equivocal act? JARA: it is an act that is capable of different interpretations. In other words, pwedeng lagyan ng kulay yung mga ginawa mo. Pwedeng may ibang kahulugan. You thought that because sobrang bait ni boylaloo/girlaloo sa iyo eh kayo na (ano yun caritas?) It is only when the thing done is equivocal that it is competent to prove the declarations accompanying it as falling within the cases of res gestae. What a person says that is explanatory of an equivocal or ambiguous act which he is then doing, or situation which he is then occupying – as that of a person in possession of property – may be proved as re s gestae. What are entries in Official Record? Entries made in official records made in the performance of his duty by a public officer of the Philippines, or by a person in the performance of a duty specially enjoyed by law. Are the items recorded in the birth certificate considered hearsay? YES, but they are considered as an exception. Why is a birth certificate trustworthy? Nobody‘s interested with the birth of a person. Jara: You might just be another bitch in this world. Furthermore, the law makes it clear that they are prima facie evidence of the facts stated therein.

Layno. Rabajante. Morana. Reyes. Suyat. Baraoidan|

43

2011-2012

[JARA POWER NOTES ON EVIDENCE]

If a mother contradicts the date alleged in the birth certificate, what should be given greater probative weight? Birth certificate. See Sec. 23, Rule 132.

d. Impressions of the emotion, behavior, condition or appearance of a person. What are the requisites for an Expert Opinion to be admissible?

When are business records excepted from the rule of hearsay evidence under the rules on Electronic Evidence? The hearsay rule is inapplicable if the following requisites are present: 1. A memorandum, report, record or data compilation of acts, events, conditions, opinions, or diagnoses, 2. Made by electronic, optical, or other similar means, 3. At or near the time of or from transmission or supply of information, 4. By a person with knowledge thereof, 5. And kept in the regular course or conduct of a business activity, 6. And such was the regular practice to make such memorandum or report, 7. All of which are shown by the testimony of the custodian or other qualified witness (Sec. 1, Rule 8, REE). OPINION AND CHARACTER EVIDENCE What is the Opinion Rule? As a rule, the opinion of a witness is inadmissible because when a witness testifies, a witness does so with respect to facts personally observed by him and it is for the court to draw conclusions from the facts testified to. Are there any exceptions? 1. The opinion of a witness regarding a matter requiring special knowledge, skill, experience or training which he is shown to possess, may be received in evidence. 2. The opinion of a witness for which proper basis is given, may be received in evidence regarding: a. The identity of a person about whom he has adequate knowledge; b. A handwriting with which he has sufficient familiarity; c. The mental sanity of a person with whom he is sufficiently acquainted;

1. That the fact to be proved is one requiring expert; 2. That the witness is really an expert. What are the Daubert and Frye standards? Frye Standard Frye involved the admissibility of opinion evidence based upon the use of an early version of the Polygraph. The D.C. Circuit Court held that scientific evidence was admissible if it was based on a scientific technique generally accepted as reliable in the scientific community. Thus, Expert Testimony was admitted based on the expert's credentials, experience, skill, and reputation. The theory was that deficiencies or flaws in the expert's conclusions would be exposed through cross-examination. This decision became known as the Frye test or the general-acceptance test. By the 1990s, the Frye test had become the majority view in federal and state courts for the admissibility of new or unusual scientific evidence, even in view of Federal Rule of Evidence 702, passed in 1975, which some courts believed to provide a more flexible test for admissibility of opinion testimony by expert witnesses. Daubert - Kumho Standard: In Daubert v. Merrell Dow Pharmaceuticals, Inc., the U.S. Supreme Court changed the standard for admissibility of expert testimony. Under Daubert, a trial judge has a duty to scrutinize evidence more rigorously to determine whether it meets the requirements of Federal Rule of Evidence 702. This rule states, "If scientific, technical, or other specialized knowledge will assist the trier of fact to understand the evidence or to determine a fact in issue, a witness qualified as an expert by knowledge, skill, experience, training, or education, may testify thereto in the form of an opinion or otherwise, if (1) the testimony is based upon sufficient facts or data, (2) the testimony is the product of reliable principles and methods, and (3) the witness has applied the principles and methods reliably to the facts of the case."

Layno. Rabajante. Morana. Reyes. Suyat. Baraoidan|

44

2011-2012

[JARA POWER NOTES ON EVIDENCE]

In Daubert, the Court stated that evidence based on innovative or unusual scientific knowledge may be admitted only after it has been established that the evidence is reliable and scientifically valid. The Court also imposed a gatekeeping function on trial judges by charging them with preventing "junk science" from entering the courtroom as evidence. To that end, Daubert outlined four considerations: testing, peer review, error rates, and acceptability in the relevant scientific community. These four tests for reliability are known as the Daubert factors or the Daubert test.

2. Let him give his factual testimony, if he has knowledge of the facts; 3. Begin the hypothetical question by asking him to assume certain facts as true; 4. Conclude the question, by, first asking the expert if he has an opinion on a certain point assuming that these facts are true and secondly, asking him, after he has answered affirmatively, to give his opinion on the point; 5. After he has stated his opinion, ask him to give his reasons.

In 1999, the U.S. Supreme Court significantly broadened that test and the trial court's gatekeeping role to include expert testimony based on technical and other specialized knowledge. Kumho Tire Co., Ltd. v. Carmichael, 526 U.S. 137, 119 S.Ct. 1167, 143 L.Ed. 2d 238 (U.S. Mar 23, 1999) (NO. 97-1709). In Kumho, the Court held that the gatekeeping obligation imposed upon trial judges by Daubert applies to scientific testimony as well as to expert opinion testimony. In order to meet its gatekeeping obligation, a trial court may use the criteria identified in Daubert only when they can be applied to determine the reliability of either the underlying scientific technique or the expert's conclusions. But inasmuch as the Daubert gatekeeping function is meant to be a flexible one, it must necessarily be tied to the particular facts of a case. Thus, the factors identified in Daubert do not constitute an exhaustive checklist or a definitive litmus test.

What is the rule on Character Evidence?

Are the said standards important in our jurisdiction?

(b) In Civil Cases:

We do not apply these standards in this jurisdiction. In the US, these standards are used with regard to the admissibility of scientific evidence. See Rule on DNA Evidence, which uses the standards to determine the probative value of DNA evidence and not to its admissibility, because in our jurisdiction admissibility is determined by the axioms of admissibility. See also the case of Rosendo C. Herrrera v. CA How do you qualify an expert witness? In Presenting an Expert Witness:

SEC. 51. Character evidence not generally admissible; exceptions:— (a) In Criminal Cases: (1) The accused may prove his good moral character which is pertinent to the moral trait involved in the offense charged. (2) Unless in rebuttal, the prosecution may not prove his bad moral character which is pertinent to the moral trait involved in the offense charged. (3) The good or bad moral character of the offended party may be proved if it tends to establish in any reasonable degree the probability or improbability of the offense charged.

Evidence of the moral character of a party in a civil case is admissible only when pertinent to the issue of character involved in the case. (c) In the case provided for in Rule 132, Section 14. What is Character in the first place? Character means the peculiar qualities impressed by nature or by habit on a person which distinguishes him from others; these constitute his real character. In other words, it means the aggregate of a person‘s traits, including those relating to care and skill and their opposites.

1. Introduce and qualify the witness;

Layno. Rabajante. Morana. Reyes. Suyat. Baraoidan|

45

2011-2012

[JARA POWER NOTES ON EVIDENCE]

In a case for Estafa, is the character of the accused relevant? See Sec. 51(a). The good character of an accused is admissible in evidence to show the improbability of his doing the act charged. The principle upon which good character maybe proved is, that it affords presumption against the commission of a crime. Why don’t we just follow the axiom of relevancy? Why do we exclude character? The evidence of a person‘s character does not prove that such person acted in conformity with such character or trait in a particular occasion. Even if it is merely circumstantial, can’t we not admit it based on the two axioms?

3. In some instances by evidence of particular acts of said person from which his character may be inferred. Can we use testimonial evidence? YES. Should the witness belong to the same community as the accused? YES. As Justice Berry said: ―As it is the fact of disposition which is important and material, there can be no reason why this fact may not be proved by any witness who knows what it is. There is certainly no reason why general repute is any better or more satisfactory evidence of disposition than the testimony of one who knows that the disposition is from his personal observation.‖

OPINION: The Rule provides for the instances where character evidence may be presented. So think. When are you going to admit character evidence?

If a person is presented to prove the common reputation, is he not offering a conclusion?

What moral trait is involved in swindling?

No. It is merely the opinion of the community.

Honesty.

Can the accused offer witness to prove that he has not defaulted in his previous transactions, and thereby prove that he did not commit estafa?

Why is the accused allowed to prove his character right away while the prosecution cannot avail of the same privilege? On the part of the prosecution, the purpose is to prevent a pronouncement of guilt not because there exist sufficient evidence of his guilt, but because he is a bad man. The reason for the rule is that evidence of bad character may create an unfair prejudice against the accused who may be convicted not because he is guilty of the crime charged, but because of his being a crooked man.

No. He cannot prove particular instances. What he must prove is his reputation as perceived by the community. BURDEN OF PROOF AND PRESUMPTIONS Burden of Proof; Definition? Burden of proof is the duty of a party to present evidence on the facts in issue necessary to establish his claim or defense by the amount of evidence required by law.

How is character evidence offered? In other words, what are the evidences that may be used in order to prove the character of a person?

What are the two concepts of burden of proof?

The best rule founded on sound logic and wise experience is that the character of a person may be proved by:

1. Burden of going forward – Party‘s obligation of producing evidence. 2. Burden of persuasion – The burden of persuading the trier of fact that the burdened party is entitled to prevail.

1. Evidence of reputation; 2. By witnesses who know him personally; and

Layno. Rabajante. Morana. Reyes. Suyat. Baraoidan|

46

2011-2012

[JARA POWER NOTES ON EVIDENCE]

Distinguish Burden of Proof from Burden of Evidence. Burden of Proof

Burden of Evidence

It is the duty of a party to present evidence on the facts in issue necessary to establish his claim or defense by the amount of evidence required by law (Sec. 1, Rule 131)

It is the duty of a party to provide evidence at any stage of the trial until he has established a prima facie case, or the like duty of the adverse party to meet and overthrow that prima facie case thus established. In both civil and criminal cases, the burden of evidence lies on the party who asserts an affirmative allegation. Shifts to the other party when one party has produced sufficient evidence to be entitled to a ruling in his favor Generally determined by the developments at the trial, or by the provisions of the substantive law or procedural rules which may relieve the party from presenting evidence on the fact alleged It creates a prima facie case and thereby sustains the said burden of evidence on the point which it covers, shifting it to the other party. It relieves those favored thereby of the burden of proving the fact presumed.

Does not shift as it remains throughout the entire case exactly where the pleadings originally placed it Generally determined by the pleadings filed by the party; and whoever asserts the affirmative of the issue has the burden of proof

It does not shift the burden of proof. However, the one who has the burden of proof is relieved from the time being, from introducing evidence in support of his averment because the presumption stands in the place of evidence.

cases, there is no need to distinguish since the accused is always presumed innocent. Reverse order of trial is possible in both civil and criminal cases. How can the reverse order of trial take place if we do not allow the court to inquire as to the defenses of the accused’s offer of a not guilty plea? If the accused raises justifying or exempting circumstances, the burden of evidence is shifted, and he must prove the existence of these circumstances. In civil cases, reverse trial is allowed when the party raises an affirmative defense. Reverse order of trial in civil cases – can the defendant stipulate in the pre-trial to change the order of trial even if he raises mere negative defenses in his pleading? Generally no. However, see Sec. 5, Rule 30 wherein the court may, for special circumstances, allow modifications in the order of trial. Can the same principle be applied in a criminal case? NO. because such would violate his right to be informed of the nature of the offense charged against him which is contained in the information. What happened in Barlin vs. Ramirez?

Why do we distinguish between negative and affirmative defenses in a civil case? Can we apply the same distinctions in a criminal case?

The defendant, Ramirez, having been appointed by the plaintiff parish priest, took possession of the church on 7/5/01. He administered if as such under the orders of his superiors until 11/14/02. His successor having been then appointed, the latter made a demand on this defendant for the delivery to him of the church, convent, and cemetery, and the sacred ornaments, books, jewels, money, and other prop. of the church. The defendant, by a written document of that date, refused to make such delivery, stating that "the town of Lagonoy, in conjunction w/ the parish priest of thereof, has seen fit to sever connection w/ the Pope at Rome and his representatives in these Islands, and to join the Filipino Church, the head of w/c is at Manila.

Because if the defendant raises affirmative defenses, he has the burden of proof. In criminal

The plaintiff brought this action against defendant, alleging in his amended complaint

Who has the burden of proof? With respect to the defendant, is it possible that he has the burden of proof? General Rule: In civil cases, the plaintiff has the burden of proof; and in criminal cases, it is the prosecution because innocence is presumed. Exception: In civil cases, burden of proof is on the defendant if he raises affirmative defenses.

Layno. Rabajante. Morana. Reyes. Suyat. Baraoidan|

47

2011-2012

[JARA POWER NOTES ON EVIDENCE]

that the Roman Catholic Church was the owner of the church bldg, the convent, cemetery, the books, money, and other property belonging thereto, and asking that it be restored to the possession thereof and that the defendant render an account of the property which he had received and which was retained by him, and for other relief. Issue: WON defendant has the right over the properties mentioned. Held: None. As to the defendant, Ramirez, it appears that he took possession of the property as the servant or agent of the plaintiff. The only right which he had to the possession at the time he took it, was the right which was given to him by the plaintiff, and he took possession under the agreement to return that possession whenever it should be demanded of him. Under such circumstances he will not be allowed, when the return of such possession is demanded by him the plaintiff, to say that the plaintiff is not the owner of the property and is not entitled to have it delivered back to him. The principle of law that a tenant can not deny his landlord‘s title, which is found in section 333, paragraph 2, of the Code of Civil Procedure, and also in the Spanish law, is applicable to a case of this kind. An answer of the defendant, Ramirez, in which he alleged that he himself was the owner of the property at the time he received it from the plaintiff, or in which he alleged that the pueblo was the owner of the property at that time, would constitute no defense. There is no claim made by him that since the delivery of the possession of the property to him by the plaintiff he has acquired the title thereto by other means, nor does he is own behalf make any claim whatever either to the property or to the possession thereof. Important matters to discuss in the case: 1. Why did the Court choose the estoppel by deed presumption? 2. Why not estoppel in pais? 3. Was there a tenant-landlord relationship in this case? What happened in Vales vs. Villa? Facts: This is an action to set aside certain transfers of real estate from the plaintiff to one of the defendants and to require that defendant to recover by good and sufficient conveyance the

title to such properties; to refund to the plaintiff a certain sum paid by plaintiff for the recovery of certain other real estate; and for an accounting by the defendants of the rents, issues and profits of certain real estate during a certain period; and for P25,000 damages. The action has been erroneously described in appellant's brief as one to annul a series of real estate transactions on the ground of duress. More correctly this is an action to compel the defendants to account unto the plaintiff for the proceeds of a series of frauds practiced upon said plaintiff. Issue: WON there is existence of fraud. Held: It is well recognized however, that a mere failure to live up to a contract is not fraudulent or deceitful. The furthest the authorities have gone along this line, and not all have gone that far, is to declare that if, at the time a contract is made, one of the parties has present in his mind the purpose and intent to break it, after getting all he can out of the other party, and that purpose and intent enter into a the contract as the main element or consideration thereof on his part, there is fraud and deceit, the authorities holding that the state of mind of the party is a fact entering into the consideration of the contract without which it would not have been made; and that, by virtue of that state of mind, the other party was deprived of property. That fact however must be alleged and proved and relied upon before it can be utilized by the person asserting its existence. It was not alleged or proved in this case and plaintiff does not rely upon it in his brief in this court. His consent was not obtained by deceit in any of the transactions. There did not exist in any one of the transactions complained of a condition where "by words and insidious machinations on the part of one of the contracting parties the other is (was) induced to execute a contract which, without them, he would not have made." Reduced to the lowest terms this action constitutes an attempt on the part of the plaintiff to extricate himself from a series of foolish transactions, if we may accept his allegations respecting them. As we have said, the sales were all made by the plaintiff with full knowledge of the facts and there appears nothing in the record which warrants a rescission of them from the standpoint of fraud. The ultimate purpose of the action is the recovery of the properties

Layno. Rabajante. Morana. Reyes. Suyat. Baraoidan|

48

2011-2012

[JARA POWER NOTES ON EVIDENCE]

described in the deed of March 22, 1909, remaining unsold. But, as appears from the evidence, the plaintiff has already recovered those properties, having purchased them from the defendants on April l4, 1913, for P6,800. Before this action was begun, therefore, plaintiff had obtained the very thing which he had been seeking to recover all through thus dealings with defendants and to obtain which he claims he had suffered so much. Having secured before this action was begun precisely what defendants had promised him and the very thing he sought, there remains nothing further to be said or done in that connection. Certainly the repurchase of the properties which he so much desired was not procured by fraud or deceit; and it was a complete termination of the relations existing between the parties arising out of the properties which he claimed were sold with a right to repurchase. After having obtained the very thing he desired and having done so in a manner which he deemed best and most suitable under the circumstances, did he not thereby terminate all relations between himself and defendants with respect to, or growing out of those properties, and can he nor repudiate not only the transaction by which he recovered them but also every other transaction which he claims related thereto? It is incomprehensible, from a legal point of view, that plaintiff, having been deprived of property by fraud and deceit, may recover that property through a voluntary agreement between him and those who deceived and defrauded him, and then repudiate not only the transaction in which he was defrauded of that property but also the very transaction by which he recovered it. Dealing with the case from the standpoint of intimidation, it should be noted of March 22, 1909, was obtained in that form by force or thereat. The validity of that conveyance is admitted; as is also the fact that the verbal agreement to reconvey was omitted from the conveyance knowingly. The claim is simply that there was a verbal agreement to reconvey on the repayment of the consideration named in the instrument and that defendants made use of the fact that the agreement was verbal and, therefore, difficult to prove, as clever by which they forced him to convey to them additional properties before they would comply with the verbal agreement. According to plaintiff's contention, then, each one of the conveyances between him and defendants subsequent to the original conveyance was an extortion, using that

word in its popular and not in its legal sense, the defendants, in order to intimidate him and thereby obtain the conveyance, threatening him with a refusal to comply with the verbal agreement to reconvey and the consequent loss of his properties. At each conveyance the defendants agreed, always verbally, as a consideration therefor, to reconvey to him the properties remaining, but each time refused to do so and proceeded, after each such conveyance, to a fresh extortion. It is contended that plaintiff, by not incorporating the verbal agreement to reconvey in the instrument itself, placed himself in a disadvantageous position; and that he executed and delivered the subsequent conveyances for the purpose of extricating himself from the unfortunate situation so produced. The ultimate extortion, the payment of P6,800 to recover the remaining properties, was the last penalty which he paid for his mistake in not incorporating the verbal agreement in the conveyance itself. All men are presumed to be sane and normal and subject to be moved by substantially the same motives. When of age and sane, they must take care of themselves. In their relation with others in the business of life, wits, sense, intelligence, training, ability and judgment meet and clash and contest, sometimes with gain and advantage to all, sometimes to a few only, with loss and injury to others. In these contests men must depend upon themselves — upon their own abilities, talents, training, sense, acument, judgment. The fact that one may be worsted by another, of itself, furnishes no cause of complaint. One man cannot complain because another is more able, or better trained, or has better sense of judgment than he has; and when the two meet on a fair field the inferior cannot murmur if the battle goes against him. The law furnishes no protection to the inferior simply because he is inferior, any more than it protects the strong because he is strong. The law furnishes protection to both alike — to one or more or less than to the other. It makes no distinction between the wise and the foolish, the great and the small, the strong and the weak. The foolish may lose all they have to the wise; but that does not mean that the law will give it back to them again. Courts cannot follow one every step of his life and extricate him from bad bargains, protect him from unwise investments, relieve him from one-sided contracts, or annul the effects of foolish acts. Courts cannot constitute themselves guardians of persons who

Layno. Rabajante. Morana. Reyes. Suyat. Baraoidan|

49

2011-2012

[JARA POWER NOTES ON EVIDENCE]

are not legally incompetent. Courts operate not because one person has been defeated or overcome by another, but because he has been defeated or overcome illegally. Men may do foolish things, make ridiculous contracts, use miserable judgment, and lose money by them — indeed, all they have in the world; but not for that alone can the law intervene and restore. There must be, in addition, a violation of law, the commission of what the law knows as an actionable wrong, before the courts are authorized to lay hold of the situation and remedy it. Furthermore, even if an actionable wrong be committed in such manner as to authorize the courts to intervene, the person injured may renounce his right to take the matter to the courts and may compromise with the wrongdoer. Or, having been placed in a very disadvantageous position by the wrong committed against him, he may be offered by his adversary one or more avenues of escape. He may be required to lose more property to his enemy or go to the courts for redress. In such case the payment of an additional sum as a means of escape is not necessarily a payment voidable for duress. The act was preceeded by an exercise of judgment. This much was plain to him: he had either to let the matter stand as it was with the loss already sustained, or go to the courts to be relieved. His judgment, operating upon this condition, told him to pay the additional sum rather than to suffer the inconvenience and expense of an action in court. A payment made under such conditions is not voidable. It is the voluntary act of a sane and mature man performed upon reflection. Not only this; it is a compromise of the original wrong and a ratification of the relation which the wrongful act was intended to establish between the parties. The same may be said with greater force of a case where a person's own voluntary act, uninfluenced by another, has put him in a disadvantageous position — a position which another may unjustly make use of to his injury. The failure to reduce a contract to writing, or to have witnesses present when a verbal agreement is made, or to record an instrument, or to exclude from the operation of its terms things verbally agreed to be excluded, etc., may place a person in a disadvantageous position with respect to another; and the demand that he pay to secure his extrication is not illegal, and a

payment made pursuant to such demand is not necessarily voidable. He pays for his lack of foresight. While the demand may be reprehensible morally, it is not illegal' and of itself is not ground for relief. There must, then, be a distinction to be made between a case where a person gives his consent reluctantly and even against his good sense and judgment, and where he, in reality, gives no consent at all, as where he executes a contract or performs an act against his will under a pressure which he cannot resist. It is clear that one acts as voluntarily and independently in the eye of the law when he acts reluctantly and with hesitation as when he acts spontaneously and joyously. Legally speaking he acts as voluntarily and freely when he acts wholly against his better sense and judgment as when he acts in conformity with them. Between the two acts there is no difference in law. But when his sense, judgment, and his will rebel and he refuses absolutely to act as requested, but is nevertheless overcome by force or intimidation to such an extent that he becomes a mere automation and acts mechanically only, a new element enters, namely, a disappearance of the personality of the actor. He ceases to exist as an independent entity with faculties and judgment, and in his place is substituted another — the one exercising the force or making use of intimidation. While his hand signs, the will which moves it is another's. While a contract is made, it has, in reality and in law, only one party to it; and, there being only one party, the one using the force or the intimidation, it is unenforceable for lack of a second party. From these considerations it is clear that every case of alleged intimidation must be examined to determine within which class it falls. If it is within the first class it is not duress in law, if it falls in the second, it is. But into whichever class it falls the party coerced may, as we have seen, waive his right to annul the contract and to recover damages. He may do this expressly or impliedly. He may expressly accept the agreement as it stands, or in a modified from, and live up to it as thus accepted. Or, he may compromise by paying something to be relieved from its effects or to have its terms changed. Or, he may accept benefits under the contract. In any one of which cases, and there are others not now necessary to mention, he renounces and waives his defense of

Layno. Rabajante. Morana. Reyes. Suyat. Baraoidan|

50

2011-2012

[JARA POWER NOTES ON EVIDENCE]

intimidation and thereby eliminates that element as one having any influence on the case thereafter. Taking the case as a whole, we are therefore of the opinion that there was no intimidation and that, if there was, the plaintiff has placed himself in a position where he was not entitled to urge it as a defense. NOTE: You must discuss what disputable presumptions were used in the case.

Is there a conclusive presumption under civil procedure? Yes. Under the Sec. 48 and 49 of Rule 39 (Res Judicata) If a judgment is not yet executory/final, can there be a presumption of regularity? YES there is a disputable presumption of correctness of judgment. How can this be rebutted?

What are the conclusive presumptions under the Rules of Court? 1. Whenever a party has, by his own declaration, act, or omission, intentionally and deliberately led another to believe a particular thing is true, and to act upon such belief, he cannot, in any litigation arising out of such declaration, act or omission, be permitted to falsify it. 2. The tenant is not permitted to deny the title of his landlord at the time of the commencement of the relation of landlord and tenant between them. Is the enumeration presumptions exclusive?

of

conclusive

No. There are many conclusive presumptions found in substantive and other procedural laws. For example, ―ignorance of the law excuses no one from compliance therewith.‖ Another is the conclusive presumeption of non-filiation in DNA cases. Review Tijam vs. Sibonghanoy. What were the conclusive presumptions applied in the case? Estoppel in pais after heavily participating in the case knowing that that the court had no jurisdiction to try the same. Give example of estoppel in pais. Note: give an example aside from Tijam

Upon appeal, ASSIGNMENT OF ERRORS. PRESENTATION OF EVIDENCE US vs. Baluyot At the general election which was held on June 6, 1916, Conrado Lerma was elected governor of the Province of Bataan. One of his competitors upon this occasion was the accused, Jose I. Baluyot, who came out third in the race. As a result of this contest a feeling of personal rancor was developed in the mind of Baluyot against his successful competitor, and during the two years which followed the accused became fully imbued with the idea that Governor Lerma was persecuting him. Because of this, Baluyot killed Gov. Lerma. During the trial of the case, it appears that after the witnesses had been examined in court for the prosecution, they were turned over to the attorney for the accused and were by him fully cross-examined. Later, when the giving of testimony for the prosecution had been concluded, the defense proceeded to introduce sundry witnesses who were examined in due course. After four had thus testified, and immediately before the accused was placed upon the stand in his own behalf, his attorney made the request that the declarations or statements made by the witnesses during the preliminary inquiry conducted by the fiscal preparatory to the prosecution of the case should be produced. The attorney for the prosecution objected on the ground that one party cannot be compelled to produced evidence in favor of the other. Issue: WON the prosecution may be compelled to produce such declarations and statements

Layno. Rabajante. Morana. Reyes. Suyat. Baraoidan|

51

2011-2012

[JARA POWER NOTES ON EVIDENCE]

made by the witnesses in a preliminary inquiry made by the fiscal. Held: No. They were not original or independent evidence of such a character as to give the accused an unqualified right to compel their production, and no proper basis was laid in the cross-examination of the witnesses who had made those statements to justify their production with a view to the impeachment of the declarants. The request was of course based upon the supposition or expectation that if the statements of the witnesses before the fiscal were produced, they might be found to contain something different from what was contained in their testimony given in court. We know of no rule of practice which sustains the contention of the appellant. The statements in question were not the sworn declarations of witnesses taken in conformity with the requirements of section 13 of General Orders, No 58, and which are commonly attached to the "expediente" transmitted by the committing magistrate to the Court of First Instance. In the case at bar the preliminary examination before the committing magistrate was waived by the accused, and the declarations of the witnesses for the prosecution were therefore not taken before the magistrate. The declarations referred to were, on the contrary, taken in an investigation conducted by the fiscal under the authority of section 1687 of the Administrative Code. This section authorizes the fiscal, if he deems it wise, to conduct an investigation into the matter of any crime or misdemeanor for the purpose of instituting or carrying on a criminal prosecution. It is expressly declared that this section shall not be construed to authorize a provincial fiscal to act as a justice of the peace in any preliminary investigation. The proceeding here contemplated is of an administrative character, and the information thereby acquired is intended for the use of the fiscal in the conduct of the prosecution. Such declarations therefore pertain to the official file in the office of the public prosecutor and are not subject to production at the mere request of the attorney for the accused where no ground therefor had been laid Qrb4Fxm. In order that we may not be misunderstood, as well as for the purpose of clarifying the practice in such matters, a few words may here by properly said in respect to the proper mode of proceeding in a case where a party wishes to

get before the court contradictory statements made by a witness who is testifying for the adversary party. For instance, if the attorney for the accused had information that a certain witness, say Pedro Gonzales, had made and signed a sworn statement before the fiscal materially different from that given in his testimony before the court, it was incumbent upon the attorney when cross-examining said witness to direct his attention to the discrepancy and to ask him if he did not make such and such statement before the fiscal or if he did not there make a statement different from that delivered in court. If the witness admits the making of such contradictory statement, the accused has the benefit of the admission, while the witness has the opportunity to explain the discrepancy, if he can. On the other hand, if the witness denies making any such contradictory statement, the accused has the right to prove that the witness did make such statement; and if the fiscal should refuse upon due notice to produce the document, secondary evidence of the contents thereof would be admissible. This process of cross-examining a witness upon the point of prior contradictory statements is called in the practice of the American courts "laying a predicate" for the introduction of contradictory statements. It is almost universally accepted that unless a ground is thus laid upon crossexamination, evidence of contradictory statements are not admissible to impeach a witness; though undoubtedly the matter is to a large extent in the discretion of the court. We wish to add that in a case of this kind, if the accused had, by affidavit or otherwise, made it appear to the satisfaction of the court that the witnesses named had made statements in their declarations before the fiscal materially at variance with their statements in court and that the production of said declarations was necessary or even desirable, in the interests of justice, the court would have had ample power to order their production. No such showing, or intimation, was made in this case; and the attorney who made the motion was merely angling at random to discover something that might prove to be favorable to his client. To put a court in error for refusing to entertain such a motion would encourage frivolous delays and tend to embarrass the speedy and proper administration of justice.

Layno. Rabajante. Morana. Reyes. Suyat. Baraoidan|

52

2011-2012

[JARA POWER NOTES ON EVIDENCE]

Under the existing rules, can the accused compel the production of the said evidence to be used in his favor? (Pwede: Right under the Constitution - Right of Compulsory process); See Rule 112, Sec. 7(b) How can an accused seek the production of evidence in court? Rule 116, Sec. 10: Upon motion of the accused showing good cause and with notice to the parties, the court, in order to prevent surprise, suppression, or alteration, may order the prosecution to produce and permit the inspection and copying or photographing of any written statement given by the complainant and other witnesses in any investigation of the offense conducted by the prosecution or other investigating officers, as well as any designated documents, documents, papers, books, accounts, letters, photographs, objects, or tangible things not otherwise privileged, which constitute or contain evidence material to any matter involved in the case and which are in the possession or under the control of the prosecution, police, or other law investigating agencies. How do you lay the predicate? By calling the attention of the witness to the former contradicting statement. If the statement be in writing they must be shown to the witness before any question is put to him. What if the witness states that he does not remember? Or what if he denies making the said statements? Offer the said evidence as an extrajudicial admission (Regalado, citing Juan Ysmael & Co vs. Hashim, 50 Phil 132). What is a leading question? A leading question is one that is framed in such a way that the question indicates to the witness the answer desired by the party asking the question. It is a question which suggests to the witness the answer which the examining party desires.

What is a misleading question? A misleading question is one which assumes as true a fact not yet testified to by the witness, or contrary to that which he has previously stated. Why is a leading question is disallowed in direct? Why is it allowed in cross? Leading questions are disallowed in direct examination because the Rules need to avoid ―coaching‖ the witness. In cross, the witness is expected to be hostile to the cross-examiner. OPINION: Again, remember the purpose of cross, hence, the cross-examiner is allowed to lead the witness or to suggest to the latter the answer the former desires in order to test whether the witness is consistent with his statement. However, the direct examiner, not being adverse to the witness, must refrain from leading his witness. The direct examiner must show that the witness knows what he is saying not as suggested by the direct examiner. Example of leading question (Trial Court setting) The Scope of Cross-examination? English Rule: where a witness is called to testify to a particular fact, he becomes a witness for all purposes and may be fully cross-examined upon all matters material to the issue, the examination not being confined to the matters inquired about in the direct examination. American Rule: restricts cross-examination to facts and circumstances which are connected with the matters that have been stated in the direct examination of the witness. What rule do we follow here? It depends on the witness and the nature of the case being heard. If the witness is an accused in the criminal case, he/she may only be cross examined on matters discussed in the direct examination. Can an accused who testifies on his on behalf, be examined using strictly the American Rule? YES, otherwise his right incrimination may be violated.

against

Layno. Rabajante. Morana. Reyes. Suyat. Baraoidan|

53

self-

2011-2012

[JARA POWER NOTES ON EVIDENCE]

If the opponent decides not to conduct a cross-examination on the witness, will that be prejudicial to the client’s cause? Can a cross-examination cause prejudice to a client? The BOSTON LEGAL Rule: Kapag walang naestablish sa direct, do not cross examine. Baka kapag nagcross-examination ka e madale ka at maestablish ang claim ng kalaban mo. What is the Rule on Revival of past recollection/present memory? 1. A witness may be allowed to refresh his memory respecting a fact, by anything written or recorded by himself or under his direction at the time when the fact occurred, or immediately thereafter, or at any other time when the fact was fresh in his memory and he knew that the same was correctly written or recorded; but in such case the writing the writing or record must be produced and may be inspected by the adverse party who may, if he chooses, crossexamine the witness upon it, and may read it in evidence. 2. A witness may testify from such a writing or record, though he retain no recollection of the particular facts,if he is able to swear that the writing or record correctly stated the transaction when made; but such evidence must be received with caution.

OFFER OF EVIDENCE How do you offer an evidence? 1. The court shall consider no evidence which has not been formally offered. The purpose for which the evidence is offered must be specified. 2. As regards the testimony of a witness, the offer must be made at the time the witness is called to testify. 3. Documentary and object evidence shall be offered after the presentation of a party‘s testimonial evidence. Such offer shall be done orally unless allowed by the court to be done in writing. Procedural steps? For testimonial evidence, just follow the codal procedure. For documentary or object evidence: 1. 2. 3. 4.

Call upon the witness; Qualify the witness; Ask the witness preliminary questions; Present evidence to witness and ask him to authenticate/identify the evidence if necessary; 5. Mark the evidence as an exhibit. 6. After all the witnesses have testified, offer the exhibits in court, identifying each of them, and stating their purpose. Challenging the qualification of a witness?

Are we to assume every time that a memorandum is available for reference?

Through ―voi dire‖ examination.

Opinion: No. The same must be presented in court.

Should you allow the witness to testify first before you disqualify him?

The memorandum from which the witness may be permitted to refresh his memory need NOT be an original writing. It is sufficient if it is shown that the witness knows the copy to be a true one, and his memory refreshed thereby enables him to testify from his own recollection of the facts, independent of his confidence in the accuracy of the copy.

No there are instances where a witness before he can be allowed to testify, may be disallowed to testify. Absolute disqualifications? Those witnesses not identified in the pre-trial brief. Are they disqualified? Generally, YES.

Layno. Rabajante. Morana. Reyes. Suyat. Baraoidan|

54

2011-2012

[JARA POWER NOTES ON EVIDENCE]

If the witness is excluded prior to testifying, what is the remedy? The Court may be liberal. Kausapin mo lang daw sabi ni Jara (in reference to Phoenix vs. Pratts). If the witness is allowed to testify, questions will be asked. In the course of propounding questions, there are certain objectionable questions. How do you classify objections? (General/Specific). Distinguish. General Objection – It does not go beyond declaring the evidence as immaterial, incompetent, irrelevant or inadmissible. In other words, it does not specify the grounds for objection. (Also known as a Broadside Objection) Specific Objection – It states why or how the evidence is irrelevant or incompetent. E.g. Objection to the question for being leading. Formal Objection – is one directed against the alleged defect in the formulation of the question. E.g. ambiguous question Substantive Objection – objections made and directed against the very nature of the evidence, i.e., it is inadmissible either because it is irrelevant or incompetent or both. e.g. parol, hearsay evidence What if there is a failure to object, can the court prohibit the witness from answering the question? Yes. If it involves an infringement of the right of the witness.

During the trial, the plaintiff, having been placed on the stand as a witness by his on attorney, testified at length and answered all the questions asked him with respect to the said contract, the details of the same, the persons who witnessed it, the place where it was made, and various other circumstances connected with its execution. These questions and answers cover six pages of the record, and yet the defendants' counsel raised no objection to the examination, aside from challenging one of the questions as leading and another of them as irrelevant. It seems that only when the examination was terminated did counsel for defendants move to strike out all of the testimony given and statements made by plaintiff in regard to the contract, on the ground that the period for the fulfillment of the contract exceeded one year and that it could not be proven except by means of a written instrument. Issue: WON the statements may be stricken out despite not being objected to during the examination of the witness. Held: No. Now then, it has been repeatedly laid down as a rule of evidence that a protest or objection against the admission of any evidence must be made at the proper time, and that if not so made it will be understood to have been waived. The proper time to make a protest or objection is when, from the question addressed to the witness, or from the answer thereto, or from the presentation of the proof, the inadmissiblity of the evidence is, or may be, inferred. A motion to strike out parol or documentary evidence from the record is useless and ineffective if made without timely protest, objection, or opposition on the part of the party against whom it was presented.

Abrenica vs. Gonda These proceedings were brought by the plaintiff to compel the defendant to return to him the two parcels of land described in the complaint which he alleges were sold by him under right of repurchase to the defendant on February 21, 1916, for the sum of P75 and for the period of seven years. The plaintiff alleged that the defendant refused to deliver said property to him when, upon the expiration of the period mentioned, he endeavored to redeem the same and tendered payment to the defendant of the sum aforesaid.

Objection to the introduction of evidence should be made before the question is answered. When no such objection is made, a motion to strike out the answer ordinarily comes too late. (De Dios Chua Soco vs. Veloso, 2 Phil. Rep., 658). The fact that the defendants' counsel asked various cross-questions, both of the plaintiff and of the other witness, in connection with the answers given by them in their direct examination, with respect to particulars concerning the contract, implies a waiver on his part to have the evidence stricken out.

Layno. Rabajante. Morana. Reyes. Suyat. Baraoidan|

55

2011-2012

[JARA POWER NOTES ON EVIDENCE]

It is true that, before cross-examining the plaintiff and one of the witnesses, this same counsel requested the permission of the court, and stipulated that his clients' rights should not be prejudiced by the answers to those witnesses in view of the motion presented to strike out their testimony; buy this stipulation of the defendants' counsel has no value or importance whatever, because, if the answers of those witnesses were stricken out, the cross-examination could have no object whatsoever, and if the questions were put to the witnesses and answered by them, they could only be taken into account by connecting them with the answers given by those witnesses on direct examination As no timely objection or protest was made to the admission of the testimony of the plaintiff with respect to the contract; and as the motion to strike out said evidence came to late; and, furthermore, as the defendants themselves, by the cross-questions put by their counsel for the witnesses in respect to said contract, tacitly waived their right to have it stricken out, that evidence, therefore, cannot be considered either inadmissible or illegal, and court, far from having erred in taking it into consideration and basing his judgment thereon, notwithstanding the fact that it was ordered to be stricken out during the trial, merely corrected the error he committed in ordering it to be so stricken out and complied with the rules of procedure hereinbefore cited. In Re Aguas? It appears from the record in this matter that on the 29th of August, 1900, during the progress of a trial then being held before the Court of First Instance at Bacolor, in the Province of Pampanga, the court had occasion to caution Angel Alberto, a witness in the case, not to look at the attorney for the defendant but to fix his attention on the judge who was at the time examining him. It seems that the witness did not give heed to this warning, and the judge thereupon arose from his seat and approaching the witness, seized him by the shoulders, and using the expression, "Lingon ang mucha" ("Look at me"), either shook him, as insisted by the attorney for the defendant, or only turned him about, as claimed by the judge and others. Whether the witness was shaken or only turned about, at all events "seizing him," brought the defendant's attorney to his feet, who, protesting against the action of the judge as coercive of the witness, demanded that a record be made of the

occurrence and that the further hearing of the case be postponed. Two days afterward the clerk entered in his record as recital of the incident substantially as above, and also a statement that on other and prior occasions the attorney, Marcelino Aguas, had been wanting in respect to the court by making use of "improper phrases," and by interrupting opposing counsel in their examination of witnesses. The court on this record adjudged the attorney to be in contempt of court and suspended him from the practice of his profession for a period of twenty days. Issue: WON the counsel may be cited in contempt of court. Held: No. In our opinion the action of the judge in seizing the witness, Alberto Angel, by the shoulder and turning him about was unwarranted and an interference with that freedom from unlawful personal violence to which every witness is entitled while giving testimony in a court of justice. Against such conduct the appellant had the right to protest and to demand were respectfully made and with due regard for the dignity of the court. The only question, therefore in this case is, Was the appellant respectful and regardful of the court's dignity in presenting his objection and asking that it be recorded in the proceedings? The witnesses say and the judge finds that "his attitude was menacing" (bastante amenazadora) in the moment of making his protest, but beyond that there is nothing in the record which even tends to show that he was disrespectful to the court or unmindful of its dignity. In our opinion both testimony and finding state a mere conclusion which, in the absence of the facts from which it was deduced, is wholly valueless to support a judgment of contempt. The statement that the attorney's attitude was "menacing" tended no more to competently establish the alleged offense of contempt than if the witnesses had testified and the court had found that his conduct was "contemptuous or lacking in respect." The specific act from which it was inferred that his attitude was menacing should have been testified to by the witnesses and found by the court, and failing that, the record does not show concrete facts sufficient to justify the conclusion that he was disrespectful to the court or offensive to its dignity.

Layno. Rabajante. Morana. Reyes. Suyat. Baraoidan|

56

2011-2012

[JARA POWER NOTES ON EVIDENCE]

US vs. Baluyot? At the general election which was held on June 6, 1916, Conrado Lerma was elected governor of the Province of Bataan. One of his competitors upon this occasion was the accused, Jose I. Baluyot, who came out third in the race. As a result of this contest a feeling of personal rancor was developed in the mind of Baluyot against his successful competitor, and during the two years which followed the accused became fully imbued with the idea that Governor Lerma was persecuting him. Because of this, Baluyot killed Gov. Lerma. During the trial of the case, it appears that after the witnesses had been examined in court for the prosecution, they were turned over to the attorney for the accused and were by him fully cross-examined. Later, when the giving of testimony for the prosecution had been concluded, the defense proceeded to introduce sundry witnesses who were examined in due course. After four had thus testified, and immediately before the accused was placed upon the stand in his own behalf, his attorney made the request that the declarations or statements made by the witnesses during the preliminary inquiry conducted by the fiscal preparatory to the prosecution of the case should be produced. The attorney for the prosecution objected on the ground that one party cannot be compelled to produced evidence in favor of the other. Issue: WON the prosecution may be compelled to produce such declarations and statements made by the witnesses in a preliminary inquiry made by the fiscal. Held: No. They were not original or independent evidence of such a character as to give the accused an unqualified right to compel their production, and no proper basis was laid in the cross-examination of the witnesses who had made those statements to justify their production with a view to the impeachment of the declarants. The request was of course based upon the supposition or expectation that if the statements of the witnesses before the fiscal were produced, they might be found to contain something different from what was contained in their testimony given in court. We know of no rule of practice which sustains the contention of the appellant. The statements

in question were not the sworn declarations of witnesses taken in conformity with the requirements of section 13 of General Orders, No 58, and which are commonly attached to the "expediente" transmitted by the committing magistrate to the Court of First Instance. In the case at bar the preliminary examination before the committing magistrate was waived by the accused, and the declarations of the witnesses for the prosecution were therefore not taken before the magistrate. The declarations referred to were, on the contrary, taken in an investigation conducted by the fiscal under the authority of section 1687 of the Administrative Code. This section authorizes the fiscal, if he deems it wise, to conduct an investigation into the matter of any crime or misdemeanor for the purpose of instituting or carrying on a criminal prosecution. It is expressly declared that this section shall not be construed to authorize a provincial fiscal to act as a justice of the peace in any preliminary investigation. The proceeding here contemplated is of an administrative character, and the information thereby acquired is intended for the use of the fiscal in the conduct of the prosecution. Such declarations therefore pertain to the official file in the office of the public prosecutor and are not subject to production at the mere request of the attorney for the accused where no ground therefor had been laid Qrb4Fxm. In order that we may not be misunderstood, as well as for the purpose of clarifying the practice in such matters, a few words may here by properly said in respect to the proper mode of proceeding in a case where a party wishes to get before the court contradictory statements made by a witness who is testifying for the adversary party. For instance, if the attorney for the accused had information that a certain witness, say Pedro Gonzales, had made and signed a sworn statement before the fiscal materially different from that given in his testimony before the court, it was incumbent upon the attorney when cross-examining said witness to direct his attention to the discrepancy and to ask him if he did not make such and such statement before the fiscal or if he did not there make a statement different from that delivered in court. If the witness admits the making of such contradictory statement, the accused has the benefit of the admission, while the witness has the opportunity to explain the discrepancy, if he can. On the other hand, if the witness denies making any such contradictory statement, the

Layno. Rabajante. Morana. Reyes. Suyat. Baraoidan|

57

2011-2012

[JARA POWER NOTES ON EVIDENCE]

accused has the right to prove that the witness did make such statement; and if the fiscal should refuse upon due notice to produce the document, secondary evidence of the contents thereof would be admissible. This process of cross-examining a witness upon the point of prior contradictory statements is called in the practice of the American courts "laying a predicate" for the introduction of contradictory statements. It is almost universally accepted that unless a ground is thus laid upon crossexamination, evidence of contradictory statements are not admissible to impeach a witness; though undoubtedly the matter is to a large extent in the discretion of the court.

Impeachment of a witness via contradictory evidence. How is this done? See Illustration made by Riano on page 325. In case of dying declarations, can you impeach the declaration of a deceased? You can now use the adverse character of the deceased declarant in impeaching the said testimony. Is the impeachment limited on the person on the stand? No.

We wish to add that in a case of this kind, if the accused had, by affidavit or otherwise, made it appear to the satisfaction of the court that the witnesses named had made statements in their declarations before the fiscal materially at variance with their statements in court and that the production of said declarations was necessary or even desirable, in the interests of justice, the court would have had ample power to order their production. No such showing, or intimation, was made in this case; and the attorney who made the motion was merely angling at random to discover something that might prove to be favorable to his client. To put a court in error for refusing to entertain such a motion would encourage frivolous delays and tend to embarrass the speedy and proper administration of justice. What is the effect when there are inconsistencies between a witness’ affidavit and his testimony in open court? Discrepancies and/or inconsistencies between a witness‘ affidavit and testimony in open court do not impair credibility as affidavits are taken ex parte and are open incomplete or inaccurate for lack or absence of searching inquiries by the investigating officer. The general rule – that contradictions and discrepancies between the testimony of a witness and his statements in an affidavit do not necessarily discredit him – is not without exception, as when the omission in the affidavit refers to a very important detail of the incident that one relating to the incident as an eyewitness would not be expected to fail to mention, or when the narration in the sworn statement substantially contradicts the testimony in court (Edwin Tabao v. People, G.R. No. 187246).

Rule on Evidence on Motions When a motion is based on facts not appearing of record, the court may hear the matter on affidavits or depositions presented by the respective parties, but the court may direct that the matter be heard wholly or partly on oral testimony or depositions. Is there a need to present evidence when one files a motion? How? Yes. Attach the evidence on the motions. Is there a motion that requires a full blown hearing and presentation of evidence? – BAIL (Evidence of Guilt is strong – quantum of proof) How about in a civil case? In cases of injunction QUANTUM OF PROOF Proof Beyond Reasonable Doubt It does not mean such degree of proof as, excluding possibility of error, produces absolute certainty. Moral certainty only is required, or that degree of proof which produces conviction in an unprejudiced mind. Clear and Convincing That degree of proof which will produce in the mind of the trier of facts a firm belief or

Layno. Rabajante. Morana. Reyes. Suyat. Baraoidan|

58

2011-2012

[JARA POWER NOTES ON EVIDENCE]

conviction as to the allegations sought to be established.

Are there Court cases that only require substantial evidence?

Is there a definition given in the Rules?

Yes. AMPARO. HABEAS DATA. KALIKASAN. (Jara: do not make the mistake in stating that substantial evidence is only available in administrative and quasi-judicial cases.

No. such quantum of proof is defined by jurisprudence. When do we use this? In extradition proceedings and other cases as may be defined by jurisprudence. Preponderance of Evidence? That which is of greater weight or more convincing than that which is offered in opposition to it. Substantial evidence That amount of relevant evidence which a reasonable mind might accept as adequate to justify a conclusion. You always expect the mind of the judge to be reasonable right? Yes. Presumption functions.

of

regularity

of

official

What is the concept of Demeanor Evidence? Whenever a witness testifies in court, the judge may be allowed to determine his credibility by observing his demeanor while he is testifying in court. What is the importance of demeanor evidence as far as the appellate court is concerned? NONE. Appellate courts are not in a position to observe the demeanor of a witness. Nevertheless, it does not preclude them from entertaining questions of fact (opinion lang po). Quasi Judicial bodies are given authority to make rules of procedure right? Are they allowed to make rules as to the quantum of evidence that may be used in their proceedings? No. Quantum of proof is always determined by the Courts. Sec. 5 Art.VIII of the Constitution is controlling.

Layno. Rabajante. Morana. Reyes. Suyat. Baraoidan|

59

View more...

Comments

Copyright ©2017 KUPDF Inc.
SUPPORT KUPDF